PPQ Flashcards by ROHIT LAWRENCE (2024)

1

Q

  1. A 2y/o has episodes of falling to the ground whilst tightening and stiffening his hands, accompanied by screaming and sweating. During the episodes the child is inconsolable but between episodes the child is well with normal development. What is the most likely diagnosis?
    a.Hyperactivity
    b.Reflex anoxic seizure
    c.Temper tantrum
    d.Tonic-clonic seizure
    e.Vasovagal syncope

A

c. Temper tantrum

Temper tantrums are common in young children and can involve intense emotional and physical reactions, including screaming, stiffening, and sweating. The child being inconsolable during the episodes but appearing well and developing normally between episodes supports this diagnosis.

How well did you know this?

1

Not at all

2

3

4

5

Perfectly

2

Q

  1. 5 y/o boy brought to GP with vague abdo pain. He has been having several loose bowel motions per day with occasional soiling of his underwear. His parents recall him passing a hard blood-streaked stool 5 days ago. Which is the most appropriate investigation?
    a.Colonoscopy with biopsies
    b.No investigations at this stage
    c.Stool microscopy, culture, and sensitivity
    d.Tissue transglutaminase antibody titres
    e.Urgent abdominal x-ray

A

b. No investigations at this stage

Initial management often includes dietary advice, hydration, and possibly the use of laxatives. If symptoms persist or worsen, further investigations may then be warranted.

How well did you know this?

1

Not at all

2

3

4

5

Perfectly

3

Q

  1. Boy born at 40 weeks with male phenotype, but neither testicl* can be palpated on examination. What is the most likely diagnosis?
    a.Androgen insensitivity syndrome
    b.Excess in utero oestrogen exposure
    c.Klinefelter’s syndrome
    d.Physiological delay in descent
    e.Retractile testes

A

d. Physiological delay in descent

In newborn boys, it is common for the testes to not be fully descended at birth. While most boys have descended testes by the time they are born, a physiological delay in descent can occur, and the testes often descend within the first few months of life. This is a more likely and common scenario compared to other conditions listed.

NOTE: Androgen insensitivity syndrome typically presents with a phenotypically female appearance despite having a 46,XY karyotype.

How well did you know this?

1

Not at all

2

3

4

5

Perfectly

4

Q

  1. 22 m/o child had cough and coryza for 2 days, and is now struggling with breathing. She has subcostal and intercostal recessions and widespread wheeze. Her temperature is 37.3, HR 125, RR 45, O2 sats 98% on room air. What is the most appropriate next step in her management?
    a.0.5L/min supplemental oxygen
    b.2 puffs beclomethasone inhaler
    c.10 puffs salbutamol inhaler
    d.Intravenous aminophylline
    e.Oral prednisolone

A

c. 10 puffs salbutamol inhaler

Given the clinical presentation of respiratory distress with wheezing, this likely represents an acute exacerbation of asthma or a similar reactive airway disease. Salbutamol (a bronchodilator) is the first-line treatment to relieve bronchospasm and improve breathing in such cases.

How well did you know this?

1

Not at all

2

3

4

5

Perfectly

5

Q

  1. 3 month old boy has 2 days of fever and crying accompanied by reduced feeding and fewer wet nappies. Temperature 38.0, HR 170, RR 38, O2 sats 97% on room air. He has reduced movement in his right leg, chest is clear, urinanalysis is normal and anterior fontanelle is slightly sunken. What is the most likely diagnosis?
    a.Developmental dysplasia of the hip
    b.Non-accidental injury
    c.Osteomyelitis
    d.Reactive arthritis
    e.Sickle cell crisis

A

c. Osteomyelitis

Osteomyelitis, an infection of the bone, is consistent with the symptoms of fever, reduced movement in the affected limb, and systemic signs of infection such as irritability and decreased feeding. The sunken fontanelle indicates dehydration, likely secondary to the fever and reduced intake. Normal urinalysis helps rule out a urinary tract infection.

How well did you know this?

1

Not at all

2

3

4

5

Perfectly

6

Q

  1. 3 m/o boy presents in acute respiratory failure and is diagnosed with Pneumocystis pneumonia. His parents are first cousins from Pakistan. What is the most likely underlying cause?
    a.Agammaglobulinaemia
    b.Chronic granulomatous disease
    c.Cystic fibrosis
    d.HIV infection
    e.Severe combined immunodeficiency

A

e. Severe combined immunodeficiency (SCID)

Pneumocystis pneumonia (PCP) is a rare but serious infection that typically occurs in individuals with significant immune deficiencies. The following points support SCID as the most likely underlying cause:

How well did you know this?

1

Not at all

2

3

4

5

Perfectly

7

Q

  1. 6 y/o boy’s parents have noticed long-standing hearing difficulties and take him to GP. Teachers have commented on poor school performance and behaviour. Speech is normal and there is no otalgia. The ear canals are narrow with non-occluding wax. Which is the most appropriate next step in his management?
    a.Aural olive oil drops
    b.Grommet insertion
    c.Pure tone audiometry
    d.Referral for hearing aids
    e.Review in 3 months

A

c. Pure tone audiometry

Pure tone audiometry is a key diagnostic test to assess the extent and nature of hearing loss. This objective assessment will help determine the severity of the hearing impairment and guide further management. Here’s why this is the best next step:

How well did you know this?

1

Not at all

2

3

4

5

Perfectly

8

Q

  1. 3 week old baby referred for prolonged jaundice, dark wet nappies, and pale stools. There are no developmental concerns and he is currently breastfed. Birth weight was 3.45kg and he currently weighs 3.2kg, Temperature is 37.1, HR 140, RR 45, O2 sats 98% on room air. Investigations:
    Hb 110 (normal)
    WCC 6 (normal)
    Platelets 430 (slightly high)
    Bilirubin 175 (high)
    Conjugated bilirubin 100 (high)
    ALP 160 (normal)
    ALT 35 (normal)
    Albumin 25 (lower end of normal)
    Sodium 137 (normal
    Potassium 4 (normal)
    What is the most appropriate next step in management?
    a.Feeding advice
    b.Inpatient ultrasound
    c.I.V. antibiotics
    d.Outpatient ultrasound
    e.Phototherapy

A

b. Inpatient ultrasound

The presence of prolonged jaundice with elevated conjugated bilirubin, pale stools, and dark urine raises concern for a biliary obstruction or other hepatobiliary pathology, such as biliary atresia. An inpatient ultrasound can help in the early diagnosis and management of these conditions, which is crucial for improving outcomes. Early identification and treatment of biliary atresia, for example, are essential for the effectiveness of surgical interventions like the Kasai procedure.

How well did you know this?

1

Not at all

2

3

4

5

Perfectly

9

Q

  1. 5 y/o boy with 2 days of fever, runny nose, cough, and sore throat. He developed a rash over the past 24 hours, temperature 39.2, HR 105, RR 30, capillary refill <2 seconds and O2 sats are 98% on room air. There is cervical lymphadenopathy, a red tongue, cracked lips, and a fine erythematous macular rash all over his body. Which is the most effective treatment?
    a.Aciclovir
    b.Aspirin
    c.Co-amoxiclav
    d.Paracetamol
    e.Phenoxymethylpenicillin

A

B. Aspirin

Given the presentation described, which strongly suggests Kawasaki disease, the most effective treatment would involve aspirin in addition to IVIG. Aspirin is used in Kawasaki disease to reduce inflammation and the risk of coronary artery complications.

How well did you know this?

1

Not at all

2

3

4

5

Perfectly

10

Q

  1. 18 m/o presents to GP with 1 day history of left ear pain. There is no discharge and mum reports normal number of wet nappies for that day. There is a bulging erythematous left tympanic membrane with no perforation. Temperature 38, HR 135, RR 33, O2 sats 98 on room air. What is the most appropriate management?
    a.Offer simple analgesia and an immediate prescription of amoxicillin
    b.Offer simple analgesia and ciprofloxacin ear drops
    c.Offer simple analgesia and delayed prescription of amoxicillin
    d.Offer simple analgesia and gentamicin ear drops
    e.Offer simple analgesia and reassurance

A

E

Acute otitis media is generally a self-limiting condition that does not require an antibiotic prescription. There are however some exceptions listed below. Analgesia should be given to relieve otalgia. Parents should be advised to seek medical help if the symptoms worsen or do not improve after 3 days.

ABx if > 4 days, younger than 2 with bilateral otitis media, otitis media with perf

If an antibiotic is given, a 5-7 day course of amoxicillin is first-line. In patients with penicillin allergy, erythromycin or clarithromycin should be given.

How well did you know this?

1

Not at all

2

3

4

5

Perfectly

11

Q

  1. A 2 y/o in GP has worsening painful rash behind the knee for 5 days. He has eczema managed with E45 cream. Temperature 37.2, HR 130, RR 30, O2 sats 98%. What is the most appropriate management?
    a.Immediate referral to Paediatric A&E
    b.Give oral Aciclovir for 1 week
    c.Give oral Flucloxacillin for 1 week
    d.Give topical dermovate
    e.Give topical fusidic acid

A

E. Give topical fusidic acid

The description of a worsening painful rash suggests a possible secondary bacterial infection, especially considering the child’s eczema, which can predispose to bacterial skin infections. Fusidic acid is an antibiotic commonly used topically for skin infections, including those caused by Staphylococcus aureus, which is often implicated in eczema-related skin infections.

How well did you know this?

1

Not at all

2

3

4

5

Perfectly

12

Q

  1. A 3 y/o boy in GP has left-sided neck lump on a background of 6 days of coryza, cough, and mild fever, but no systemic symptoms. Temperature 37.0, HR 105, capillary refill <1s, RR 26, O2 sats 99% on room air. ENT exam is normal and chest is clear. He has a single enlarged lymph node in the left posterior chain. What is the most appropriate next step in management?
    a.Arrange urgent ENT referral
    b.Arrange USS
    c.Arrange urgent FBC
    d.Prescribe Amoxicillin 250mg TDS for 7 days
    e.Reassurance and watchful waiting

A

e. Reassurance and watchful waiting

In a child with a recent upper respiratory tract infection (coryza, cough, mild fever) and a single enlarged lymph node in the posterior cervical chain, especially without systemic symptoms or concerning findings on examination, a conservative approach with reassurance and watchful waiting is often appropriate.

The likely diagnosis in Reactive cervical lymphadenopathy

How well did you know this?

1

Not at all

2

3

4

5

Perfectly

13

Q

  1. 7 y/o boy in GP with bedwetting for 2 months having previously been dry at night since age 4. Abdo exam is normal. What is the most appropriate initial investigation?
    a.Abdominal USS
    b.Abdominal x-ray
    c.Finger prick glucose test
    d.Renal function
    e.Urine dipstick

A

e. Urine dipstick

Bedwetting (nocturnal enuresis) in a previously dry child may have various causes, including urinary tract infections, diabetes, constipation, or emotional stress. However, the most common cause is primary nocturnal enuresis, which is more likely if the child has never been dry at night consistently.

How well did you know this?

1

Not at all

2

3

4

5

Perfectly

14

Q

  1. 35 y/o man presents to A&E agitated and requesting medication. He has mild tremor, cool and clammy skin, and goosebumps (piloerection). Ambulance staff found him in the street with an empty bottle of cider. Temperature 36.8, BP 140/100, HR 100. What is the most likely diagnosis?
    a.Alcohol intoxication
    b.Alcohol withdrawal syndrome
    c.Opiate intoxication
    d.Opiate withdrawal syndrome
    e.Tricyclic antidepressant intoxication

A

b. Alcohol withdrawal syndrome

Alcohol withdrawal syndrome typically occurs in individuals with a history of heavy or prolonged alcohol use who suddenly reduce or cease their alcohol intake. Common symptoms include agitation, tremors, autonomic hyperactivity (such as cool and clammy skin, piloerection), and in severe cases, hallucinations, seizures, or delirium tremens.

Given the presentation of agitation and physical signs such as tremor and autonomic hyperactivity, along with the history of alcohol consumption, alcohol withdrawal syndrome is the most likely diagnosis in this case.

How well did you know this?

1

Not at all

2

3

4

5

Perfectly

15

Q

  1. 20 y/o medical student attends A&E on the eve of an exam with difficulty breathing, palpitations, and chest tightness. She says she feels close to collapse, and feels like she is going to die. This has happened before and she was prescribed Propranolol but this is no longer effective, and she has routinely declined psychological interventions. Routine blood tests, ECG, and CXR are all normal. What is the most appropriate next step in her management?
    a.Diazepam
    b.Fluoxetine
    c.Mirtazapine
    d.Risperidone
    e.Venlafaxine

A

B. Fluoxetine

Fluoxetine is a selective serotonin reuptake inhibitor (SSRI) antidepressant commonly used in the treatment of panic disorder and other anxiety disorders. It is effective in reducing the frequency and severity of panic attacks and can provide long-term relief when taken regularly.

How well did you know this?

1

Not at all

2

3

4

5

Perfectly

16

Q

  1. 30 y/o white woman is on routine anti-psychotic medication for schizophrenia and has the following blood result:
    Hb 140 (normal)
    WCC (low)
    Platelets 200 (normal)
    Neutrophils 1.2 (low)
    Lymphocytes 1.3 (normal)
    Total cholesterol 5.3 (high)
    Total cholesterol: HDL ratio 4.6 (high)
    HbA1C 40 (normal)
    What medication is she most likely taking?
    a.Aripiprazole
    b.Clozapine
    c.Haloperidol
    d.Quetiapine
    e.Risperidone

A

B. Clozapine

Clozapine is an atypical antipsychotic medication commonly used in the treatment of schizophrenia, particularly in cases where other antipsychotic medications have been ineffective or poorly tolerated.

The blood results provided, specifically the low white cell count (WCC), low neutrophil count, and normal lymphocyte count, are consistent with potential hematological side effects of clozapine, such as agranulocytosis or neutropenia. Regular monitoring of blood counts is essential when using clozapine due to the risk of these adverse effects.

How well did you know this?

1

Not at all

2

3

4

5

Perfectly

17

Q

  1. 21 y/o woman admitted to psych ward for an acute episode. She has been prescribed Amisulpride for the past 3 weeks and is now worries she is pregnant because she has missed her period, however her pregnancy test is negative. Which blood test should be requested in addition to routine bloods?
    a.Luteinising hormone
    b.Progesterone
    c.Prolactin
    d.Thyroid stimulating hormone
    e.Triglycerides

A

c. Prolactin

Amisulpride is an antipsychotic medication that can increase prolactin levels by blocking dopamine receptors in the brain. Elevated prolactin levels can lead to amenorrhea (missed periods) and galactorrhea (milk production from the breasts) in women.

Therefore, checking serum prolactin levels can help assess whether the missed period is due to elevated prolactin levels induced by Amisulpride. If prolactin levels are found to be elevated, this could explain the amenorrhea, and appropriate management or adjustment of medication can be considered.

How well did you know this?

1

Not at all

2

3

4

5

Perfectly

18

Q

  1. 15 y/o girl has 12 months of irritable mood, lack of pleasure, poor sleep, poor concentration, and low appetite. She is struggling to concentrate at school and is getting into trouble for low marks. What is the most appropriate first-line treatment?
    a.Cognitive analytic therapy
    b.Cognitive behavioural therapy
    c.Family therapy
    d.Fluoxetine
    e.Sertraline

A

d. Fluoxetine

Fluoxetine is a selective serotonin reuptake inhibitor (SSRI) antidepressant commonly used in the treatment of major depressive disorder (MDD) in adolescents. It is one of the first-line pharmacological treatments for depression in this age group, as recommended by clinical guidelines.

How well did you know this?

1

Not at all

2

3

4

5

Perfectly

19

Q

  1. 19 y/o man with 1 day of fever, muscle stiffness, palpitations, and difficulty breathing. He has a history of psychotic depression for which he has been taking Fluoxetine for 6 weeks and Risperidone for 1 week. He is agitated with temperature of 39.8, HR 114, BP 172/89, and RR 30. Chest examination is otherwise clear, and neuro exam is normal except for globally increased tone. Which investigation would be most useful diagnostically?
    a.CT head
    b.ECG
    c.EEG
    d.Lumbar puncture
    e.Serum creatine kinase

A

e. Serum creatine kinase (CK)

The presentation is suggestive of a potential serotonin syndrome, a potentially life-threatening condition characterized by excessive serotonin activity in the central nervous system. Serotonin syndrome can occur as a result of the interaction between serotonergic medications such as Fluoxetine and Risperidone, leading to symptoms such as fever, muscle rigidity, agitation, and autonomic instability.

Serum CK levels can be elevated in serotonin syndrome due to the muscular hyperactivity and breakdown associated with the condition. Elevated CK levels can help support the diagnosis of serotonin syndrome and differentiate it from other conditions presenting with similar symptoms.

How well did you know this?

1

Not at all

2

3

4

5

Perfectly

20

Q

  1. 23 y/o man admitted for 3rd person auditory hallucinations, irritability, and beliefs that his neighbours are watching him. This is the first time he has experienced these symptoms and they have lasted for 3 weeks. What is the most likely diagnosis?
    a.Acute and transient psychotic disorder
    b.Adjustment disorder
    c.Paranoid personality disorder
    d.Schizoaffective disorder
    e.Schizophrenia

A

a. Acute and transient psychotic disorder

Acute and transient psychotic disorder is characterized by the sudden onset of psychotic symptoms, such as hallucinations, delusions, disorganized thinking, or grossly disorganized or catatonic behavior, lasting for at least a day but less than a month. This diagnosis is appropriate when the symptoms do not meet the criteria for other psychotic disorders like schizophrenia.

Given the brief duration of symptoms (less than a month) and the absence of a clear history of recurrent psychotic episodes or chronicity, acute and transient psychotic disorder is the most likely diagnosis in this case.

How well did you know this?

1

Not at all

2

3

4

5

Perfectly

21

Q

  1. 15 y/o girl in A&E after fainting at school. BMI is 16, she has been restricting calorie intake for 1 year, she exercises every day and feels she is overweight. Which additional feature would be needed for a diagnosis of anorexia nervosa?
    a.Bradycardia
    b.Fear of gaining weight
    c.Laxative abuse
    d.Muscle weakness
    e.Thinning of hair on head

A

b. Fear of gaining weight

An essential diagnostic criterion for anorexia nervosa is a persistent fear of gaining weight or becoming fat, despite being underweight. This fear may manifest in various behaviors, including restrictive eating, excessive exercise, and avoidance of calorie-dense foods.

How well did you know this?

1

Not at all

2

3

4

5

Perfectly

22

Q

  1. 45 y/o man presents with worsening tremor for a few weeks. Has a history of schizophrenia in remission for which he is taking Haloperidol. There is bilateral tremor and cog-wheel rigidity in upper limbs. What is the most appropriate treatment for his symptoms?
    a.Co-beneldopa
    b.Entacapone
    c.Pramipexole
    d.Procyclidine hydrochloride
    e.Selegiline hydrochloride

A

d. Procyclidine hydrochloride

The presentation of bilateral tremor and cog-wheel rigidity suggests parkinsonism, a movement disorder characterized by symptoms similar to those of Parkinson’s disease. Haloperidol, an antipsychotic medication, can induce extrapyramidal symptoms such as parkinsonism as a side effect.

Procyclidine hydrochloride is an anticholinergic medication commonly used to alleviate extrapyramidal symptoms induced by antipsychotic medications like Haloperidol. It works by blocking the action of acetylcholine in the central nervous system, which can help reduce tremor, rigidity, and other parkinsonian symptoms.

How well did you know this?

1

Not at all

2

3

4

5

Perfectly

23

Q

  1. 84 y/o man presents with gradual deterioration in memory for 3 years. He forgets details of conversations and to attend doctor’s appointments, though he can still manage his finances and cook and clean for himself. He regularly goes to the local shops and never gets lost. Which part of the brain is likely to be affected?
    a.Cerebellum
    b.Frontal lobe
    c.Occipital lobe
    d.Parietal lobe
    e.Temporal lobe

A

e. Temporal lobe

The temporal lobes play a crucial role in memory formation and retrieval, particularly episodic memory, which involves the recollection of specific events and experiences. Damage or degeneration in the temporal lobes can lead to difficulties in forming new memories and retrieving past memories, resulting in symptoms such as forgetfulness and memory loss.

Given that the individual is experiencing memory problems but still retains functional abilities related to daily living (such as cooking, cleaning, managing finances, and navigating familiar environments), it suggests that the memory impairment is selective and not globally disabling. This pattern of impairment is often seen in conditions affecting the temporal lobes, such as Alzheimer’s disease, which is the most common cause of age-related memory decline and dementia.

How well did you know this?

1

Not at all

2

3

4

5

Perfectly

24

Q

  1. A 25 y/o man has tried Olanzapine and Haloperidol but continues to experience symptoms of schizophrenia. What is the most appropriate next step in his management?
    a.Aripirazole
    b.Clonazepam
    c.Clozapine
    d.Lithium
    e.Quetiapine

A

Clozapine

Clozapine is considered the gold standard for treatment-resistant schizophrenia. It has been shown to be effective in patients who have not responded to other antipsychotic medications. However, it is typically reserved as a second-line or third-line option due to its potential for serious side effects, including agranulocytosis (a significant reduction in white blood cell count), which requires regular monitoring.

How well did you know this?

1

Not at all

2

3

4

5

Perfectly

25

Q

  1. 25 y/o woman has recurrent episodes of sudden onset sweating, dry mouth, “butterflies” in her stomach, difficulty breathing, and fear of impending death. These symptoms have been occurring twice weekly for the past 2 years. What is the most likely diagnosis?
    a.Dissociative disorder
    b.Generalised anxiety disorder
    c.Hypochondriasis
    d.Panic disorder
    e.Somatisation disorder

A

Panic disorder

Panic disorder is characterized by recurrent and unexpected panic attacks, which are sudden episodes of intense fear or discomfort that reach a peak within minutes. The symptoms described in the scenario, including sweating, dry mouth, gastrointestinal discomfort (“butterflies” in the stomach), difficulty breathing, and fear of impending doom, are typical features of panic attacks.

The presence of recurrent panic attacks over a period of at least several months, accompanied by persistent worry about having additional attacks, or significant behavioral changes related to the attacks, is consistent with the diagnosis of panic disorder.

While other conditions such as generalized anxiety disorder (Option b) involve excessive worry and anxiety, panic disorder specifically involves recurrent panic attacks characterized by sudden and intense fear.

How well did you know this?

1

Not at all

2

3

4

5

Perfectly

26

Q

  1. 19 y/o woman with OCD is prescribed Sertraline at the maximum dose but it doesn’t control her symptoms. She has been unable to attend university for 4 months, and CBT has not helped in the past. What is the most appropriate next step in her management?
    a.Add another antidepressant
    b.Change to an antidepressant of another class
    c.Refer for additional and more intensive CBT
    d.Refer for deep brain stimulation
    e.Refer for ECT

A

B. Change to an antidepressant of another class

When a patient with OCD does not respond adequately to one selective serotonin reuptake inhibitor (SSRI) such as Sertraline, switching to another antidepressant of a different class, such as a tricyclic antidepressant (TCA) or a serotonin-norepinephrine reuptake inhibitor (SNRI), may be considered.

How well did you know this?

1

Not at all

2

3

4

5

Perfectly

27

Q

  1. 25 y/o man attends A&E with headache. He is speaking rapidly and smiling as he talks. He scratches his skin constantly and claims to feel insects crawling up him. His temperature is 36.5, HR 130, and BP 154/84. What is the most likely diagnosis?
    a.Alcohol withdrawal
    b.Benzodiazepine withdrawal
    c.Cannabis intoxication
    d.Cocaine intoxication
    e.Heroin withdrawal

A

d. Cocaine intoxication

Cocaine is a stimulant drug that can cause a range of physiological and psychological effects, including euphoria, increased energy, rapid speech, tactile hallucinations (such as the sensation of insects crawling on or under the skin, known as formication), and heightened sympathetic activity leading to elevated heart rate and blood pressure.

Scratching behavior, often described as “cocaine bugs” or “cocaine psychosis,” is a common phenomenon associated with cocaine use, where individuals experiencing tactile hallucinations may scratch or pick at their skin in an attempt to alleviate the sensation of insects crawling on or under the skin.

How well did you know this?

1

Not at all

2

3

4

5

Perfectly

28

Q

  1. 45 y/o man is taken to A&E by police who are concerned for his health after being in custody for 20 hours. He is known to consume alcohol to excess, and appears dishevelled, agitated, and is tremulous. Blood tests are unremarkable except for moderately deranged LFTs. What is the most likely diagnosis?
    a.Alcohol withdrawal syndrome
    b.Alcohol induced hepatitis
    c.Hepatic encephalopathy
    d.Opioid withdrawal syndrome
    e.Wernicke’s encephalopathy

A

Alcohol withdrawal syndrome

Alcohol withdrawal syndrome typically occurs in individuals with a history of chronic alcohol abuse who suddenly decrease or stop their alcohol consumption. Symptoms may include agitation, tremors, anxiety, sweating, nausea, vomiting, insomnia, and occasionally hallucinations or seizures.

The presentation described, including agitation, tremors, and elevated LFTs, is consistent with alcohol withdrawal syndrome. The tremulousness and agitation suggest autonomic hyperactivity, which is characteristic of alcohol withdrawal.

How well did you know this?

1

Not at all

2

3

4

5

Perfectly

29

Q

  1. 58 y/o man attends addiction services for initial alcohol dependence assessment. He regularly attends A&E following overdose attempts. He requests detoxification. He lives alone with few social contacts and has poorly controlled asthma. What is the most appropriate next step in management?
    a.CBT
    b.Community detoxification
    c.Disulfiram
    d.Inpatient detoxification
    e.Motivational interviewing

A

d. Inpatient detoxification

Inpatient detoxification offers a structured and supervised environment where the patient can receive medical care and support during the withdrawal process. This setting is particularly important for individuals with severe alcohol dependence and those at risk of complications such as overdose attempts, as it allows for close monitoring of symptoms and medical management of any complications that may arise.

Inpatient detoxification also provides an opportunity for comprehensive assessment, including evaluation of the patient’s asthma and optimization of asthma management during detoxification.

How well did you know this?

1

Not at all

2

3

4

5

Perfectly

30

Q

  1. 30 y/o man attends A&E with palpitations, dizziness, and chest pain – he is worried that he has had a heart attack. The chest pain is sharp and variable in location, and he gives a history of 6 months of low mood following being laid off from work. He has attended multiple local hospitals for several years and has reported a variety of different symptoms including headaches over the right ear, difficulty swallowing, and griping stomach pains, but says “the doctor’s haven’t been able to find out what’s wrong with me”. His ECG is normal. What is the most likely diagnosis?
    a.Borderline personality disorder
    b.Factitious disorder
    c.Generalised anxiety disorder
    d.Panic disorder
    e.Somatisation disorder

A

e. Somatization disorder

Somatization disorder is characterized by the presence of multiple physical symptoms that cannot be explained by a medical condition. These symptoms often lead to frequent medical visits and investigations, as the individual believes them to be indicative of a serious illness despite medical reassurance. The symptoms are typically varied and may involve different organ systems.

The patient’s history of chest pain, dizziness, palpitations, headaches over the right ear, difficulty swallowing, and griping stomach pains, along with normal findings on ECG and a history of low mood following job loss, are consistent with the diagnosis of somatization disorder.

How well did you know this?

1

Not at all

2

3

4

5

Perfectly

31

Q

  1. A 35 y/o man with Fragile X syndrome lives alone, does voluntary work, and travels independently to familiar places, though he needs help planning travel to new areas. He attended a special needs school and has a carer supporting him by helping him to pay his bills. What is his most likely IQ?
    a.20
    b.40
    c.65
    d.80
    e.110

A

65

Individuals with Fragile X syndrome typically exhibit a wide range of intellectual abilities, but the majority fall within the mild to moderate range of intellectual disability. An IQ of around 65 would be consistent with the level of independence described in the scenario, where the individual can perform some tasks independently but may require support with more complex activities and decision-making.

How well did you know this?

1

Not at all

2

3

4

5

Perfectly

32

Q

  1. 48 y/o man has hypertension, anxiety, gout, and back pain. His routine ECG shows a QT interval of 460ms (380-440 normal range). Which is the most likely causative medication?
    a.Allopurinol
    b.Amitriptyline
    c.Co-codamol
    d.Diazepam
    e.Ramipril

A

b. Amitriptyline

Amitriptyline is a tricyclic antidepressant (TCA) known to prolong the QT interval. Prolongation of the QT interval can predispose individuals to a type of arrhythmia called torsades de pointes, which can be life-threatening.

How well did you know this?

1

Not at all

2

3

4

5

Perfectly

33

Q

  1. A 29 y/o woman in GP is complaining of palpitations, sweating, dry mouth, and paraesthesia each time she leaves the house. She has a PMHx of asthma. Which is the most appropriate drug class to use?
    a.Atypical anti-psychotic
    b.Benzodiazepine
    c.Beta blocker
    d.Sedating antihistamine
    e.SSRI

A

Beta blocker

These symptoms are suggestive of anxiety or panic attacks, especially given their occurrence in specific situations (leaving the house), and beta blockers are commonly used to manage the physical symptoms associated with anxiety or panic disorders. Beta blockers can help alleviate symptoms such as palpitations, sweating, and tremors by blocking the effects of adrenaline on the heart and peripheral vasculature, thus reducing sympathetic nervous system activity.

How well did you know this?

1

Not at all

2

3

4

5

Perfectly

34

Q

  1. 38 y/o woman in GP has 8 months of amenorrhea on a background of T2DM and BPAD. Her pregnancy test is negative. Investigations:
    Luteal LH 2.0 (normal)
    Luteal FSH 2.0 (normal)
    TSH 2.3 (normal)
    Testosterone 1.0 (normal)
    SHBG 80.0 (normal)
    Prolactin 821 (high)
    Which medication is most likely to have caused this?
    a.Diazepam
    b.Metformin
    c.Risperidone
    d.Sertraline
    e.Zopiclone

A

c. Risperidone

Risperidone is an atypical antipsychotic medication commonly used to treat conditions like bipolar affective disorder. It is known to increase prolactin levels through its dopamine receptor-blocking effects, particularly at higher doses. Elevated prolactin levels can lead to amenorrhea or irregular menstrual cycles in women due to the suppression of gonadotropin-releasing hormone (GnRH) from the hypothalamus, which in turn inhibits ovulation.

How well did you know this?

1

Not at all

2

3

4

5

Perfectly

35

Q

  1. 28 y/o woman attends GP for 6 week post-natal check following emergency C-section. She is feeling tired, low in energy, has poor appetite, and is having some frightening thoughts or about hurting her baby. Which is the most likely diagnosis?
    a.Adjustment disorder
    b.Baby blues
    c.PTSD
    d.Post-natal depression
    e.Puerperal psychosis

A

d. Post-natal depression

Post-natal depression is a mood disorder that can occur after childbirth, typically within the first few weeks to months. It is characterized by symptoms such as low mood, tearfulness, irritability, fatigue, changes in appetite, and difficulty bonding with the baby. Frightening thoughts about harming the baby can also occur in post-natal depression, although it’s important to note that these thoughts are distressing to the mother and are not acted upon

How well did you know this?

1

Not at all

2

3

4

5

Perfectly

36

Q

  1. 24 y/o seen in GP for 6 months of low mood, poor concentration, and trouble sleeping. He denies suicidal ideation or self-harm. He denies drinking excessively, smoking at all, and using recreational drugs. He is a waiter and is struggling at work and has a PHQ-9 of 6 (mild severity). What is the most appropriate course of action?
    a.Advise mindfulness classes
    b.Refer for CBT
    c.Start Citalopram
    d.Start Fluoxetine
    e.Write a fit note (medical certificate) for 4 weeks

A

Advise mindfulness classes

Mindfulness-based interventions, such as mindfulness classes, have been shown to be effective in reducing symptoms of depression, particularly in individuals with mild-to-moderate depression. They provide techniques for managing stress, enhancing self-awareness, and promoting emotional regulation, which can be beneficial for individuals experiencing low mood and other depressive symptoms.

How well did you know this?

1

Not at all

2

3

4

5

Perfectly

37

Q

  1. 29 y/o woman visits GP with heavy sensation in her vagin* when exercising. She is aware of a bulge in her vagin*, sometimes associated with a desire to urinate. She had a vagin*l delivery 2 years ago. O/E there is some laxity of the anterior vagin*l wall but this does not descend to the introitus on straining. Urine culture is negative. What is the most appropriate management plan?
    a.Oxybutynin hydrochloride
    b.Pelvic floor exercises
    c.Refer for urodynamic testing
    d.Refer to urogynaecology clinic
    e.USS of pelvis

A

b. Pelvic floor exercises

Pelvic floor exercises, also known as Kegel exercises, are commonly recommended as the first-line treatment for mild to moderate symptoms of pelvic organ prolapse, which appears to be the likely diagnosis in this case. These exercises aim to strengthen the muscles supporting the pelvic organs and can help improve symptoms of pelvic floor dysfunction, including sensations of heaviness or bulging in the vagin* and urinary symptoms

How well did you know this?

1

Not at all

2

3

4

5

Perfectly

38

Q

  1. 42 y/o woman is 34 weeks pregnant and has had repeat admissions for severe abdominal pain. She currently needs opiate analgesia but is still struggling with the pain. USS shows a sub-serosal fibroid measuring 7cm x 9cm. What is the best management option for her pain?
    a.Continuous regular opiates
    b.Epidural anaesthesia
    c.NSAIDs
    d.Remifentanyl patient-controlled analgesia
    e.Uterine artery embolisation

A

NSAIDs

Nonsteroidal anti-inflammatory drugs (NSAIDs) can be considered as the best management option for her pain in this situation. NSAIDs are effective for managing pain associated with fibroids and are relatively safe to use during pregnancy, especially in the third trimester. However, they should be avoided earlier in pregnancy due to potential risks to the fetus, such as premature closure of the ductus arteriosus and oligohydramnios.

39

Q

  1. 33 y/o woman 32 weeks pregnant has glycosuria for the second time. Fundal height is 36cm and the fetal parts are difficult to palpate. Her routine glucose testing at 27 weeks was normal. What is the most appropriate next step in her management?
    a.Advise low glycaemic index diet
    b.Perform 75g glucose tolerance test
    c.Send HbA1c
    d.Start Insulin
    e.Start Metformin

A

b. Perform 75g glucose tolerance test

The presence of glycosuria, increased fundal height, and difficulty palpating fetal parts suggest the possibility of gestational diabetes mellitus (GDM) or macrosomia (large fetal size). While routine glucose testing at 27 weeks was normal, the development of glycosuria at 32 weeks raises concerns for glucose intolerance that may have developed later in pregnancy.

Performing a 75g glucose tolerance test (GTT) would help to accurately diagnose gestational diabetes mellitus (GDM) and guide appropriate management. This test is the gold standard for diagnosing GDM and involves measuring blood glucose levels at fasting and at specific time intervals after a glucose load.

How well did you know this?

1

Not at all

2

3

4

5

Perfectly

40

Q

  1. 40 y/o woman is successfully treated for ectopic pregnancy with methotrexate. After what interval may she safely conceive again?
    a.1 year
    b.3 months
    c.6 months
    d.Her next menses
    e.Immediately

A

c. 6 months

Waiting for at least 6 months after methotrexate treatment for an ectopic pregnancy before attempting to conceive again is often recommended. This allows time for the methotrexate to clear from the body and reduces the risk of potential complications or adverse effects on a subsequent pregnancy.

How well did you know this?

1

Not at all

2

3

4

5

Perfectly

41

Q

  1. A woman is having a repeat USS at 32 weeks gestation for a previously low-lying placenta. The USS shows the placenta is clear of the cervical os and an additional succenturiate lobe is seen. Which condition needs to be excluded in the USS?
    a.Cervical shortening
    b.Fetal growth restriction
    c.Placenta praevia
    d.Vasa praevia
    e.Velamentous cord insertion

A

d. Vasa praevia

Vasa praevia is a rare but potentially life-threatening condition where fetal blood vessels, unprotected by Wharton’s jelly, run across the internal os of the cervix or between the placental lobes. This condition can lead to vessel rupture and severe fetal hemorrhage during labor or rupture of membranes.

While the presence of a low-lying placenta (placenta previa) was previously a concern, the current ultrasound showing the placenta clear of the cervical os eliminates this as a primary concern. The additional finding of a succenturiate lobe indicates the presence of an accessory placental lobe, which is typically not associated with significant risks.

How well did you know this?

1

Not at all

2

3

4

5

Perfectly

42

Q

  1. 63 y/o woman presents with a 2 week history of post-menopausal vagin*l bleeding. BMI is 39.2. Pelvic USS shows irregular endometrial thickening (14mm). Outpatient endometrial sampling fails because of cervical stenosis. What is the most appropriate management?
    a.CT abdomen and pelvis
    b.Hysteroscopy with dilatation and curettage
    c.Repeat USS in 2 weeks
    d.Total abdominal hysterectomy + bilateral salpingo-oopherectomy
    e.Total laparoscopic hysterectomy + bilateral salpingo-oopherectomy

A

b. Hysteroscopy with dilatation and curettage

Hysteroscopy with dilatation and curettage (D&C) is the most appropriate next step in this situation. It allows direct visualization of the endometrial cavity and enables targeted biopsy or removal of any suspicious tissue. Additionally, cervical stenosis can often be managed during hysteroscopy by dilating the cervix, allowing for successful sampling or biopsy.

How well did you know this?

1

Not at all

2

3

4

5

Perfectly

43

Q

  1. 32 y/o nulliparous woman is 8 weeks pregnant and experiencing severe nausea and vomiting that makes it impossible for her to work. She is taking 50mg cyclizine OD (once daily) with no effect. What is the best next management option?
    a.Admit for I.V. fluids
    b.Change to prochlorperazine
    c.Increase frequency of cyclizine to TDS (three times daily)
    d.Add a PPI
    e.Add Prochlorperazine

A

B. Change to prochlorperazine

Switching to prochlorperazine (Option b) is indeed a reasonable next step in the management of severe nausea and vomiting (hyperemesis gravidarum) in pregnancy if cyclizine is ineffective. Prochlorperazine is another commonly used antiemetic medication that may be more effective for some individuals.

How well did you know this?

1

Not at all

2

3

4

5

Perfectly

44

Q

  1. 29 y/o woman with past history of cocaine and cannabis use denies using for the past 3 years. Which additional test should be done for this patient at booking?
    a.Hair toxicology screen
    b.Hepatitis C
    c.Hepatitis E
    d.Serum toxicology screen
    e.Urine toxicology screen

A

e. Urine toxicology screen

Urine toxicology screening is a common method used to detect recent drug use. It can detect the presence of various substances, including cocaine and cannabis, typically within a few days to a week after use, depending on factors such as the frequency and amount of drug use.

How well did you know this?

1

Not at all

2

3

4

5

Perfectly

45

Q

  1. 33 y/o woman has Ventouse delivery at 39 weeks gestation and requires manual removal of placenta. The removal is uncomplicated with an EBL of 600ml. She plans to mix feed her baby for the first 6 months. When can she expect her menstrual cycle to return?
    a.Cannot be predicted
    b.When she is more than 50% bottle feeding
    c.When she stops breastfeeding
    d.Within 6 months
    e.Within 6 weeks

A

a. Cannot be predicted

After a vagin*l delivery, the return of menstruation can vary from woman to woman, and it is influenced by factors such as breastfeeding, hormonal changes, and individual physiology.

In the case of mixed feeding, where the mother is both breastfeeding and supplementing with formula, the return of menstruation can be more variable compared to exclusive breastfeeding. Breastfeeding typically delays the return of menstruation due to the suppression of ovulation caused by the hormone prolactin.

Given that the woman plans to mix feed her baby for the first 6 months, it’s likely that she may experience a delay in the return of her menstrual cycle compared to if she were exclusively bottle feeding. However, the return of menstruation cannot be predicted with certainty, as it varies among individuals.

How well did you know this?

1

Not at all

2

3

4

5

Perfectly

46

Q

  1. 20 y/o woman attends A&E. She realised 4 days after her menses finished that she had forgotten to remove her tampon. She has now removed it and is asymptomatic but concerned about the possible consequences. What is the most appropriate management?
    a.Advise vagin*l douching
    b.Arrange USS pelvis
    c.Perform high vagin*l swabs
    d.Prescribe prophylactic antibiotics
    e.Reassure and discharge

A

e. Reassure and discharge

If the woman has removed the forgotten tampon and is asymptomatic, there is typically no need for further intervention

How well did you know this?

1

Not at all

2

3

4

5

Perfectly

47

Q

  1. 60 y/o woman presents with discomfort during sex and vagin*l dryness. What is the best management option?
    a.Commence oral HRT
    b.Commence SSRI
    c.Commence topical vagin*l oestrogen
    d.Commence transdermal HRT
    e.Commence vagin*l lubricants

A

c. Commence topical vagin*l estrogen

Topical vagin*l estrogen therapy is often considered the first-line treatment for symptoms of vagin*l dryness and discomfort during intercourse in postmenopausal women. It works by replenishing estrogen levels in the vagin*l tissues, restoring moisture and elasticity, and improving symptoms of vagin*l atrophy.

How well did you know this?

1

Not at all

2

3

4

5

Perfectly

48

Q

  1. 17 y/o girl has delayed menarche and short stature. Investigations:
    FSH 70 (very high)
    LH 40 (very high)
    Which investigation is likely to produce a definitive diagnosis?
    a.Karyotyping
    b.MRI scan of pituitary fossa
    c.Serum oestradiol
    d.Thyroid function tests
    e.Pelvic USS

A

a. Karyotyping

Karyotyping can help identify any chromosomal abnormalities, such as Turner syndrome (45,X), which is a common cause of POI in young females. Turner syndrome is associated with short stature, delayed puberty, and ovarian dysgenesis.

How well did you know this?

1

Not at all

2

3

4

5

Perfectly

49

Q

  1. 40 y/o woman is having a normal vagin*l delivery with planned active management of third stage. She has a history of essential hypertension. Which is the most appropriate drug to reduce risk of post-partum haemorrhage?
    a.Dinoprostone (prostaglandin E2)
    b.Ergometrine maleate
    c.Labetalol hydrochloride
    d.Magnesium sulphate
    e.Oxytocin (Syntocinon)

A

e. Oxytocin (Syntocinon)

Oxytocin is the preferred uterotonic agent for preventing postpartum hemorrhage during the third stage of labor. It is a synthetic form of the natural hormone oxytocin, which stimulates uterine contractions and helps to prevent excessive bleeding after delivery by promoting the contraction of the uterine muscles and reducing the risk of uterine atony.

How well did you know this?

1

Not at all

2

3

4

5

Perfectly

50

Q

  1. 32 y/o woman has diagnostic laparoscopy to investigate pelvic pain, and has a catheter inserted in recovery for urinary retention. The catheter is removed the next morning. When can she be safely discharged home?
    a.When she has voided more than 200ml
    b.Straight away with advice to return if she has problems voiding
    c.When her post-void residual volume is 0ml
    d.When her post-void residual volume is 100ml
    e.When her post-void residual volume is 300ml

A

c. When her post-void residual volume is 0ml

How well did you know this?

1

Not at all

2

3

4

5

Perfectly

51

Q

  1. 25 y/o woman who is 38 weeks pregnant is seen in triage for irregular painful contractions lasting 6 hours. She took paracetamol 6 hours ago. She is examined and found to be 2cm dilated. Which analgesia is most appropriate?
    a.Co-dydramol
    b.Epidural analgesia
    c.Nitrous oxide
    d.NSAID
    e.Warm bath

A

b. Epidural analgesia

Epidural analgesia is commonly used during labor to provide effective pain relief while allowing the woman to remain awake and alert. It can provide continuous pain relief throughout labor and delivery, making it suitable for women experiencing prolonged or intense pain during labor.

How well did you know this?

1

Not at all

2

3

4

5

Perfectly

52

Q

  1. 41 y/o women attends her dating scan. LMP dates make her 12 weeks pregnant. An intrauterine pregnancy is seen with no fetal heartbeat. CRL is equivalent to a fetus of 9 weeks gestation. What is the best next step?
    a.Admit her for laparoscopy
    b.Counsel her on management options for miscarriage
    c.Offer her a repeat USS in 1 week
    d.Offer treatment with methotrexate
    e.Serum beta-hCG now and repeat in 48 hours

A

b. Counsel her on management options for miscarriage.

In this case, the woman should be informed about the diagnosis of a missed miscarriage and offered options for management, which may include expectant management (allowing the miscarriage to occur naturally), medical management (using medication to induce miscarriage), or surgical management (such as dilation and curettage). The choice of management option should be based on the woman’s preferences, clinical condition, and any medical indications.

How well did you know this?

1

Not at all

2

3

4

5

Perfectly

53

Q

  1. 37 y/o woman has her anomaly scan at 20+3 weeks of pregnancy, and it reveals echogenic bowel. What is the most likely cause?
    a.Duodenal atresia
    b.Incidental finding
    c.Male gender
    d.Trisomy 18
    e.Trisomy 21

A

b. Incidental finding.

Echogenic bowel can sometimes be seen as an incidental finding on ultrasound and may not necessarily indicate a specific abnormality or pathology. However, it can also be associated with certain conditions such as infections, genetic abnormalities (such as trisomy 21 or 18), fetal bowel obstruction (like duodenal atresia), or other gastrointestinal issues. While trisomy 21 can sometimes present with echogenic bowel, trisomy 18 is more strongly associated with this finding.

How well did you know this?

1

Not at all

2

3

4

5

Perfectly

54

Q

  1. 29 y/o woman had had intermittent vagin*l bleeding since insertion of the Mirena coil 6 weeks ago. What is the most appropriate investigation?
    a.Cervical smear
    b.Colposcopy
    c.High vagin*l swab
    d.No investigation required
    e.Transvagin*l pelvic USS

A

e. Transvagin*l pelvic ultrasound (USS)

Transvagin*l pelvic ultrasound would help assess the position of the Mirena coil, ensuring it’s properly placed within the uterine cavity and ruling out any complications, such as uterine perforation or abnormal positioning, which could be causing the vagin*l bleeding. Additionally, it can help evaluate the endometrial lining and identify any other potential causes of the bleeding. Therefore, this investigation is crucial in determining the cause of the symptoms and guiding further management.

How well did you know this?

1

Not at all

2

3

4

5

Perfectly

55

Q

  1. A woman with sickle cell trait is 8 weeks into an unplanned pregnancy. Which investigation should be offered first?
    a.Chorionic villus sampling
    b.Free fetal DNA testing
    c.Partner FBC
    d.Partner haemoglobin electrophoresis
    e.Partner karyotyping

A

d. Partner haemoglobin electrophoresis.

This investigation is essential because if the partner is found to have sickle cell trait or another hemoglobinopathy, it could increase the risk of the unborn child inheriting a more severe form of the disease, such as sickle cell disease or another hemoglobinopathy. Identifying the partner’s hemoglobin status allows for informed genetic counseling and consideration of further testing or interventions to assess the risk to the fetus.

How well did you know this?

1

Not at all

2

3

4

5

Perfectly

56

Q

  1. A Rh negative woman has light vagin*l bleeding and pain at 9 weeks gestation. USS shows a viable 9 week intrauterine pregnancy. The bleeding resolves 48 hours later. What further treatment does she require?
    a.250IU Anti-D prophylaxis immediately
    b.Kleihauer test
    c.No further action
    d.Progesterone pessaries
    e.Repeat scan in 2 weeks

A

c. No further action.

Since the woman’s Rh status is negative and there are no signs of miscarriage or complications following the bleeding episode, further treatment, such as Anti-D prophylaxis or progesterone pessaries, is not indicated at this time.

How well did you know this?

1

Not at all

2

3

4

5

Perfectly

57

Q

  1. 24 y/o woman has urge incontinence causing low mood. She previously had a cl*toridectomy aged 7 years old in Somalia. She is nulliparous. She has two younger brothers. What is the most appropriate next step in her management?
    a.Referral to FGM clinic
    b.Referral to mental health team
    c.Referral to police
    d.Referral to social services
    e.Referral to uro-gynaecology

A

A. Referral to FGM clinic

How well did you know this?

1

Not at all

2

3

4

5

Perfectly

58

Q

  1. 36 y/o woman sees GP for routine appointment at 32 weeks pregnancy. This is her first pregnancy and has so far been uncomplicated. She has normal fetal movements and is generally well with temperature 36.3, HR 90, BP 128/82, RR 13, O2 sats 96% on room air. Fundal height is 32cm and fetal HR is normal. Urine dipstick shows ++ protein but no other abnormalities. What is the most appropriate management?
    a.Immediate referral to obstetrics
    b.Repeat urinanalysis in 1 day
    c.Repeat urinanalysis in 1 week
    d.Send urine for MC&S and start Nitrofurantoin
    e.Send urine for MC&S and start Trimethoprim

A

a. Immediate referral to obstetrics.

Proteinuria in pregnancy can be an indication of various conditions, including preeclampsia. Preeclampsia is a serious condition characterized by hypertension and proteinuria after 20 weeks of gestation. Given the presence of proteinuria along with the gestational age of 32 weeks, immediate referral to obstetrics for further evaluation and management is warranted to assess for preeclampsia and ensure the well-being of both the woman and the fetus.

How well did you know this?

1

Not at all

2

3

4

5

Perfectly

59

Q

  1. 19 y/o woman in GP has a 3 week history of post-coital vagin*l bleeding. She had a Nexplanon (Progesterone-only subdermal implant) inserted 18 months ago and has been amenorrhoeic for the last 12 months. vagin*l and speculum exams are normal and urine pregnancy test is normal. What is the most appropriate next investigation?
    a.Blood test for clotting screen
    b.Cervical cytology
    c.Endocervical, chlamydial, and high vagin*l swabs
    d.Transvagin*l USS
    e.Urine for microscopy, culture, and sensitivities

A

C. Endocervical, chlamydial, and high vagin*l swabs

How well did you know this?

1

Not at all

2

3

4

5

Perfectly

60

Q

  1. 19 y/o woman attends GP for contraception review. She started Microgynon-30 14 months ago and reports a recent migraine without aura which lasted 2 days (which she has never experienced before). She is currently a smoker but has no other CVS disease or VTE risk factors. Her temperature is 36.5, HR 70, BP 115/63, RR 14, O2 sats 97% on room air, and her BMI is 21. What is the most appropriate next step in management?
    a.Continue Microgynon-30 and offer smoking cessation
    b.Continue Microgynon-30 and start Sumitriptan
    c.Stop Microgynon-30 and offer Microgynon-20
    d.Stop Microgynon-30 and offer progesterone-only contraception
    e.Stop Microgynon-30 and start NuvaRing

A

d. Stop Microgynon-30 and offer progesterone-only contraception.

Migraine with aura is a contraindication to combined hormonal contraceptives (CHCs), such as Microgynon-30, due to the increased risk of ischemic stroke. While the woman’s recent migraine was without aura, the presence of any migraine headache while using CHCs is still considered a precaution, especially in combination with smoking, which further increases the risk of vascular events.

Therefore, the most appropriate action would be to discontinue Microgynon-30 and offer progesterone-only contraception, which does not carry the same increased risk of ischemic stroke in women with migraine headaches, even in the absence of aura. This would help minimize the risk of adverse cardiovascular events while still providing effective contraception.

How well did you know this?

1

Not at all

2

3

4

5

Perfectly

61

Q

  1. 26 y/o woman has 24 hours of vulval rash and painful urination. What is the most appropriate treatment?
    a.Oral aciclovir
    b.Oral flucloxacillin
    c.Topical aciclovir
    d.Topical clotrimazole
    e.Topical fusidic acid

A

a. Oral aciclovir.

The symptoms are suggestive of genital herpes, a common sexually transmitted infection characterized by painful genital ulcers and rash, along with dysuria. Oral aciclovir is the standard treatment for genital herpes and can help reduce the severity and duration of symptoms, speed up healing of lesions, and decrease the risk of recurrent outbreaks.

How well did you know this?

1

Not at all

2

3

4

5

Perfectly

62

Q

  1. 29 y/o seen in GP wanting to conceive. She has epilepsy and her last seizure was 18 months ago. She currently has a copper IUD in situ and is taking sodium valproate 1000mg BD. What is the most appropriate management?
    a.Leave IUD in situ, continue sodium valproate, and refer her for specialist advice
    b.Remove IUD, continue sodium valproate 1000mg BD and start folic acid 5mg OD
    c.Remove IUD, stop sodium valproate 1000mg and start carbamazepine 600mg BD and folic acid 5mg OD
    d.Remove ID, stop sodium valproate, and start folic acid 400mcg OD

A

b. Remove IUD, continue sodium valproate 1000mg BD, and start folic acid 5mg OD.

While planning for pregnancy, it’s essential to consider both the woman’s epilepsy management and the potential impact of antiepileptic drugs (AEDs) on pregnancy outcomes. Sodium valproate is associated with an increased risk of congenital malformations and neurodevelopmental disorders in babies born to mothers taking it during pregnancy, especially at higher doses.

However, abruptly stopping sodium valproate can increase the risk of seizures, which can also be harmful during pregnancy. Therefore, the recommendation is typically to continue sodium valproate if it is the most effective treatment for controlling seizures, but to use the lowest effective dose and add folic acid supplementation at a higher dose (5mg daily) to reduce the risk of neural tube defects in the baby.

How well did you know this?

1

Not at all

2

3

4

5

Perfectly

63

Q

  1. 31 y/o woman in GP is 34 weeks pregnant and presents with dysuria, frequency, and urgency. Temperature 37.2, HR 80, BP 126/78, RR 18, O2 sats 99% on room air. She has mild suprapubic pain with no rebound or tenderness. What is the most appropriate initial management plan?
    a.Arrange clinical review in 24 hours
    b.No investigation required, treat empirically with antibiotics
    c.Reassure, no treatment needed
    d.Send urine for MC&S and treat empirically with antibiotics without waiting for results
    e.Send urine for MC&S and treat with antibiotics based on the results

A

d. Send urine for MC&S (Microscopy, Culture, and Sensitivity) and treat empirically with antibiotics without waiting for results.

Urinary tract infections (UTIs) are common during pregnancy and can have serious implications if left untreated. Pregnant women are at higher risk of complications from UTIs, including pyelonephritis and preterm labor.

Empirical treatment with antibiotics without waiting for culture results is typically recommended in pregnant women with symptoms suggestive of a UTI, especially in the third trimester, to promptly alleviate symptoms and reduce the risk of complications.

How well did you know this?

1

Not at all

2

3

4

5

Perfectly

64

Q

  1. A baby is born with 45XO karyotype. What is the most likely cardiac abnormality associated with this syndrome?

A

Bicuspid aortic valve

How well did you know this?

1

Not at all

2

3

4

5

Perfectly

65

Q

  1. 15 y/o boy has worsening severe headaches for 1 month and has been vomiting in the morning. There is no history of trauma and he is afebrile. What definitive investigation should now be requested?

A

Brain MRI

How well did you know this?

1

Not at all

2

3

4

5

Perfectly

66

Q

  1. 5 y/o boy has a systolic murmur which is soft and audible at the left sternal edge. The boy is asymptomatic. What is the most likely diagnosis?

A

Innocent murmur

How well did you know this?

1

Not at all

2

3

4

5

Perfectly

67

Q

  1. Child has symptoms of perennial rhinitis (blocked nose, nasal voice, running eyes). What is the most likely causative allergen in this scenario?

A

Dust mites

How well did you know this?

1

Not at all

2

3

4

5

Perfectly

68

Q

  1. 2 y/o girl has a fever of 39.7 and a painful, swollen right eye with preserved eye movements (see picture below). What is the most appropriate first-line treatment?

A

Oral co-amox

How well did you know this?

1

Not at all

2

3

4

5

Perfectly

69

Q

  1. 15 y/o girl weighs 100kg. She has striae and darker skin in her armpits and neck. She has been feeling tired and has had episodes of vulval thrush, but is otherwise well. A blood test shows a fasting blood glucose of 7.6mmol/L (3-6 normal range). What pharmacological treatment should be considered?

A

Metformin

How well did you know this?

1

Not at all

2

3

4

5

Perfectly

70

Q

  1. Mother with SLE has a baby who is born with a slow heartbeat. What is the most likely diagnosis?

A

Congenital heart block

How well did you know this?

1

Not at all

2

3

4

5

Perfectly

71

Q

  1. 2 y/o boy has suddenly started crying and intermittently pulling his legs up. He has vomited several times, and the last vomitus was green. He has not opened his bowels for several hours. He is resuscitated appropriately. What is the definitive management for this condition?

A

Rectal air insufflation

How well did you know this?

1

Not at all

2

3

4

5

Perfectly

72

Q

  1. 13 y/o girl seen in GP with fever, sore throat, and cervical lymphadenopathy. Her temperature is 37.9 and she has white exudates on enlarged tonsils. She is prescribed Amoxicillin 500mg TDS, then 7 days later re-presents with a widespread rash. What is the most likely causative organism?

A

Epstein-Barr virus

How well did you know this?

1

Not at all

2

3

4

5

Perfectly

73

Q

  1. 2 week old boy in GP has bilateral red eyes with purulent discharge since birth. What is the most likely causative organism?

A

Chlamydia trachomatis

How well did you know this?

1

Not at all

2

3

4

5

Perfectly

74

Q

  1. 20 y/o woman presents to A&E with agitation, confusion, high temperature, sweating, muscle twitches, and diarrhoea. She was recently started on medication for depression. What is the most likely diagnosis?

A

Serotonin syndrome

How well did you know this?

1

Not at all

2

3

4

5

Perfectly

75

Q

  1. 60 y/o man with schizophrenia is taking haloperidol and has developed uncontrolled repetitive movements of his jaw with grimacing and protrusion of his tongue. What phenomenon is this?

A

Tardive dyskinesia

How well did you know this?

1

Not at all

2

3

4

5

Perfectly

76

Q

  1. 97 y/o woman has dementia and frequently tries to leave her nursing home an 03:00 in her nightgown to feed the swans. What legal framework may be considered in order to prevent her from leaving?

A

Deprivation of Liberty Safeguards

How well did you know this?

1

Not at all

2

3

4

5

Perfectly

77

Q

  1. 27 y/o woman reviewed by community mental health team for schizophrenia follow-up. She describes experiences where she thinks she isn’t real. What phenomenon is this?

A

Depersonalisation

How well did you know this?

1

Not at all

2

3

4

5

Perfectly

78

Q

  1. 36 y/o man started on lithium for BPAD. He is scheduled to have regular monitoring of lithium levels, renal function, calcium, and glucose. Which other blood test should be monitored?

A

TFTs

How well did you know this?

1

Not at all

2

3

4

5

Perfectly

79

Q

  1. 50 y/o man believes wife is cheating on him, gathering evidence by repeated obsessive examination of her underwear. What is the most likely psychiatric diagnosis?

A

Othello syndrome

How well did you know this?

1

Not at all

2

3

4

5

Perfectly

80

Q

  1. 40 y/o woman attends A&E after her mother’s funeral with sudden onset blindness, but seems largely unconcerned about this new development. Physical examination and investigations are normal. What is the most likely disorder?

A

Conversion disorder

How well did you know this?

1

Not at all

2

3

4

5

Perfectly

81

Q

  1. 20 y/o man with schizophrenia tells his support worker that the TV news presenter talks about him. What psychotic phenomenon is he describing?

A

Ideas of reference

How well did you know this?

1

Not at all

2

3

4

5

Perfectly

82

Q

  1. 56 y/o man brought to A&E by ambulance after he’s found collapsed in a park. His pupils are constricted and his RR is 6. Which mediation should be urgently administered?

A

Nalaxone

How well did you know this?

1

Not at all

2

3

4

5

Perfectly

83

Q

  1. 78 y/o woman has 2 weeks of progressive lethargy, nausea, and dizziness. She has depression and has recently increased her dose of Citalopram. What is the most likely biochemical abnormality?

A

Hyponatraemia

How well did you know this?

1

Not at all

2

3

4

5

Perfectly

84

Q

  1. 22 y/o female with diagnosis of OCD presents to her GP for a repeat prescription of her medication. Which class of drug is first line for OCD?

A

SSRI

How well did you know this?

1

Not at all

2

3

4

5

Perfectly

85

Q

  1. 40 y/o doctor has central chest pain but a normal ECG. 3 months ag he attended rapid access chest pain clinic but was discharged after normal exercise stress test. His only PMHx is chronic rhinitis. What is the most likely drug of abuse in this case?

A

Cocaine

How well did you know this?

1

Not at all

2

3

4

5

Perfectly

86

Q

  1. 22 y/o woman has 6 months of restricted eating, amenorrhea, social anxiety, and low mood. She has fine hair on her face and her BMI is 16.8. Her observations are otherwise normal. What is the most appropriate first-line pharmacological intervention?

A

SSRI (Fluoxetine)

How well did you know this?

1

Not at all

2

3

4

5

Perfectly

87

Q

  1. 16 y/o attends GP with difficulties at college: she fears sitting on public transport and feels scared to leave her house. She avoids the supermarket and now does her grocery shopping online. What is the most likely diagnosis?

A

Social phobia

88

Q

  1. 22 y/0 woman is planning a water birth. She is low risk with no medical problems. What complication of normal delivery is increased by a water birth?

A

Infection

89

Q

  1. 80 y/o woman with a BMI of 25, COPD, and angina complains of back pain and a dragging sensation in her vagin*. She has a uterine prolapse and a normal pelvic USS. What is the most appropriate treatment option?

A

Pelvic floor muscle training

90

Q

  1. 8 y/o girl brought to gynaecology clinic with offensive vagin*l discharge, but is otherwise well. What is the most likely cause?

A

Foreign body

91

Q

  1. 49 y/o woman has heavy, irregular menstrual bleeding. TVUSS shows a thickened endometrium with cystic spaces. What investigation would be diagnostic in this scenario?

A

Biopsy

92

Q

  1. 32 y/o with mild asthma has unexplained intra-uterine death at 37 weeks gestation. She is induced and has epidural analgesia and a spontaneous vagin*l delivery. She is very distressed and would like to go home. What should be given to prevent her from expressing breast milk in this scenario?

A

Cabergoline

93

Q

  1. 21 y/o woman is 5 weeks pregnant with abdominal pain. USS shows no evidence of an intrauterine pregnancy, a corpus luteum in the left ovary, and otherwise normal adenexae. Her serum b-hCG is 700 (<18 normal range). How should this pregnancy be classified?

A

Pregnancy of Unknown Location

94

Q

  1. A low risk woman would like to have epidural anaesthesia but is concerned about the risks. What is the risk of permanent paralysis (give as 1:X)

A

1 in 100,000 to 1 in 250,000 epidurals administered

95

Q

  1. 31 y/o woman is 37 weeks pregnant and attends GP with itchy rash, though she is systemically well. Temperature 36.8, HR 80, BP 116/78, RR 18, O2 sats 99% on room air. She has the rash depicted below. What is the most likely diagnosis?

A

Pruritic urticarial papules and plaques of pregnancy (PUPPP)

96

Q

  1. 19 y/o woman with intra uterine device in situ presents to GP with new onset low abdominal pain and spotting. Temperature 36.3, HR 83, BP 110/90, RR 18, O2 sats 98% on room air. What is the most appropriate initial investigation?

A

Urinary B-HcG

97

Q

  1. 34 y/o woman in GP is 28 weeks pregnant. She had normal antenatal screening and scans, and her pregnancy is uncomplicated. She has already had the influenza vaccination. Which other immunisation does she require?

A

Whooping cough

98

Q

  1. 28 y/o woman attends GP requesting emergency contraception 5 days after unprotected intercourse. What is the most effective option available to her?

A

Copper IUD

99

Q

  1. 38 y/o woman in GP requests contraception. She has severe menorrhagia and had a gastric sleeve bypass 12 months ago. What is the first line contraceptive management in this scenario?

A

Progestogen-only intrauterine system

100

Q

  1. A 13 year old girl in the Paediatric Emergency Department has drowsiness, weight loss and tiredness over the last 10 days. Her capillary blood gas results on arrival shows:
    pH 7.01 (7.35 - 7.45)
    PCO2 2.9 kPa (4.6 - 6.4)
    HCO3 9 mmol/L (22 - 30)
    Glucose 24 mmol/L (3.0 - 6.0)
    Blood ketones 6 mmol/L

Following an initial ABC assessment, bloods were taken. Her latest observations are: temperature 37C, pulse rate 118 bpm, BP 105/74 mmHg, capillary refill time 3 seconds, respiratory rate 27 breaths per minute, oxygen saturation 96% breathing air.

Rank the following subsequent management steps in order of priority with (1) being performed first and (5) being performed last
a.SC insulin
b.Repeat blood gas
c.Administer a fluid bolus
d.IV insulin
e.Calculate IV fluid requirement

A

a.SC insulin 5
b.Repeat blood gas 4
c.Administer a fluid bolus 1
d.IV insulin 3
e.Calculate IV fluid requirement 2

101

Q

A previously well, fully immunised, 2 year old girl, has a fever, irritability and vomiting for the last 3 days. A lumbar puncture is performed, and her CSF results are as follows:
CSF protein 0.8 g/L (0.15 - 0.45)
Glucose 2.9 mmol/L (2.2 - 2.4) (blood glucose 7 mmol/L)
Polymorphonuclear cells 95%
Mononuclear cells 5%

Rank the following causative organisms in order of likelihood with (1) being the most likely and (5) being the least likely.
a.Streptococcus pneumoniae
b.Cryptococcus
c.Haemophilus influenzae type B
d.Staphylococcus aureus
e.Enterovirus

A

a.Streptococcus pneumoniae 1
b.Cryptococcus 4
c.Haemophilus influenzae type B 2
d.Staphylococcus aureus 3
e.Enterovirus 5

102

Q

. A 2 week old baby has been recently diagnosed with trisomy 21. Rank the following cardiac defects in order of likelihood
a.ASD
b.Pulmonary atresia
c.Mild coarctation of the aorta
d.AVSD
e.VSD

A

a.ASD 3
b.Pulmonary atresia 5
c.Mild coarctation of the aorta 4
d.AVSD 1
e.VSD 2

103

Q

A 4 month old boy in General Practice has 2 months of slowly worsening vomiting after feeding. He frequently cries and arches his back after feeding and refuses feeds. He is formula fed. He was just below the 50th centile for weight at birth and is now just above the 25th percentile.

Rank the following interventions according to the sequence in which they should be implemented with (1) being carried out first and (5) being carried out last.
a.Refer to paediatrics team
b.Trial smaller and more frequent feeds
c.Trial of proton pump inhibitor treatment
d.Trial of feed thickener
e.Prescribe alginate therapy with feeds

A

a.Refer to paediatrics team 5
b.Trial smaller and more frequent feeds 1
c.Trial of proton pump inhibitor treatment 4
d.Trial of feed thickener 2
e.Prescribe alginate therapy with feeds 3

104

Q

A 32 year old woman with bipolar affective disorder is in the 11th week of her first pregnancy. She has not been taking medication for the last 3 months and wants to remain medication free throughout the pregnancy and postnatal period.

Rank the following disorders in order of likelihood of occurrence in this woman with (1) being the most likely and (5) being the least likely.
a.Post-traumatic stress disorder
b.Puerperal psychosis
c.Postnatal depression
d.Alcohol use disorder
e.Generalised anxiety disorder

A

a.Post-traumatic stress disorder 5
b.Puerperal psychosis 2
c.Postnatal depression 1
d.Alcohol use disorder 4
e.Generalised anxiety disorder 3

105

Q

A 21 year old woman in the Emergency Department states that she is going to kill herself because the voices in her head are telling her to do so. Her feelings reportedly intensified following an argument with her mother earlier in the day. She demands to be admitted to the mental health unit, saying ‘it will all be your fault when I commit suicide”. She has a history of cutting and is well known to mental health services from previous crisis presentations.

Rank the following differential diagnoses in order of likelihood, with (1) being the most likely and (5) being the least likely.
a.Dissocial personality disorder
b.Paranoid schizophrenia
c.Depressive episode of moderate severity
d.Severe depressive episode with psychotic features
e.Emotionally unstable personality disorder

A

a.Dissocial personality disorder 5
b.Paranoid schizophrenia 4
c.Depressive episode of moderate severity 3
d.Severe depressive episode with psychotic features 1
e.Emotionally unstable personality disorder 2

106

Q

A 37 year old woman in the Emergency Department has low mood, insomnia, an inability to concentrate and hopelessness for the past 6 weeks. She recently lost her job as a corporate lawyer. She had an episode of depression at the age of 26 years, which was treated with fluoxetine for 6 months. She reports being a social drinker (average consumption is 2 glasses of wine per week).

Rank the following risk factors for depression in order of significance for this woman, with (1) being the most significant and (5) being the least significant.
a.Alcohol intake
b.Gender
c.Age
d.Previous depressive episode
e.Recent unemployment

A

A 37 year old woman in the Emergency Department has low mood, insomnia, an inability to concentrate and hopelessness for the past 6 weeks. She recently lost her job as a corporate lawyer. She had an episode of depression at the age of 26 years, which was treated with fluoxetine for 6 months. She reports being a social drinker (average consumption is 2 glasses of wine per week).

Rank the following risk factors for depression in order of significance for this woman, with (1) being the most significant and (5) being the least significant.
a.Alcohol intake 5
b.Gender 3
c.Age 4
d.Previous depressive episode 1
e.Recent unemployment 2

107

Q

. The son of an 81 year old man in General Practice is concerned about his father. His father lives alone and has become more forgetful over the past 3 months. He has left his house without shutting the door several times and has been getting lost on the way to the shops. He has well-controlled hypertension and diet-controlled Type 2 Diabetes Mellitus.

Rank the following management steps according to the sequence in which they should be implemented with (1) being carried out first and (5) being carried out last.
a.Arrange joint consultation with father and son
b.Refer to local Memory Clinic
c.Perform an assessment of cognition with a cognitive assessment tool
d.Arrange confusion screen blood tests
e.Assess for acute reversible causes of confusion

A

a.Arrange joint consultation with father and son 1
b.Refer to local Memory Clinic 5
c.Perform an assessment of cognition with a cognitive assessment tool 3
d.Arrange confusion screen blood tests 4
e.Assess for acute reversible causes of confusion 2

108

Q

A 24 year old woman presents with a 2 day history of worsening right-sided lower abdominal pain. She is sexually active and normally uses condoms. She has a 28 day cycle and her last period was 6 weeks ago. One day ago she had loose stools and attributed this to a bad take away. She has nausea and loss of appetite but no vomiting. Her temperature is 37.3C, pulse 96 bpm, BP 110/64 mmHg, respiratory rate 18 breaths per minute, oxygen saturation 98% breathing room air.

Urine dip: blood 1+ only
Blood test results: Hb 85 g/L (115 - 150), CRP 10 mg/L (<5)

Rank the following diagnoses in order of likelihood with (1) being the most likely and (5) being the least likely.
a.Appendicitis
b.Urinary tract infection
c.Pyelonephritis
d.Ectopic pregnancy
e.Ovarian cyst accident

A

a.Appendicitis 3
b.Urinary tract infection 5
c.Pyelonephritis 4
d.Ectopic pregnancy 1
e.Ovarian cyst accident 2

109

Q

A 25 year old woman in the Emergency Department is 8 days post delivery. She was induced at 38 weeks’ gestation for pre-labour rupture of membranes. She had a forceps delivery for foetal distress and a postpartum haemorrhage of 700mls. She was discharged on day 4 post delivery. She says she is finding it hard to cope, worries about the baby all the time and is unable to sleep because she feels she needs to watch the baby. She cries every day and is worried someone will take the baby from her. She is concerned her breast milk is inadequate. She has been sweating at night but has not checked her temperature. She also has a constant headache.

Rank the following differential diagnoses in order of likelihood with (1) being the most likely and (5) being the least likely.
a.Postnatal depression
b.Sepsis
c.Schizophrenia
d.Baby blues
e.Pre-eclampsia

A

a.Postnatal depression 2
b.Sepsis 3
c.Schizophrenia 4
d.Baby blues 1
e.Pre-eclampsia 5

110

Q

A 36 year old mother of three young children is not using any contraception. She had a recent unplanned pregnancy and opted for a termination. She occasionally experiences headaches and smokes five cigarettes per day. Her BMI is 30 kg/m2. She has been in a relationship for 18 months.

Rank the following contraceptive choices for this woman in order of appropriateness, with (1) being the most appropriate and (5) being the least appropriate.
a.Combined oral contraceptive pill
b.Nexplanon implant
c.Sterilisation
d.Mirena IUS
e.Condoms

A

a.Combined oral contraceptive pill 4
b.Nexplanon implant 2
c.Sterilization 5
d.Mirena IUS 1
e.Condoms 3

111

Q

A heterosexual couple in their early 30s have been trying to conceive for more than 1 year without success. Neither has any children.

Rank the following potential causes of their infertility in order of likelihood with (1) being the most likely and (5) being the least likely.
a.Gamete defects
b.Male infertility
c.Uterine or peritoneal disorders
d.Tubal damage
e.Ovulatory disorders

A

a.Gamete defects 5
b.Male infertility 2
c.Uterine or peritoneal disorders 3
d.Tubal damage 4
e.Ovulatory disorders 1

112

Q

A 9 year old girl is in Paediatric Outpatients following parental concerns about her weight. During the consultation the girl reveals she is being bullied at school. Her weight is 49kg, and height is 1.37m (given growth chart with this plotted). Her thyroid function test results are as follows. TSH 3.5mU/L (0.7-4.1), free T4 16 pmol/L (12-22). Which is the most appropriate next step in her weight management?
a.Prescribe metformin
b.Prescribe orlistat
c.Refer to a paediatric dietician
d.Refer to Child and Adolescent Mental Health Services (CAMHS)
e.Refer to endocrinology

A

Refer to a paediatric dietician

113

Q

A 3 week old girl has had loose stools since day 2 of life and there are now streaks of blood in the stool. The baby examies well with normal observations, normal findings on abdominal examination, but there is dry skin on the scalp and face. Which is the most likely diagnosis?
a.Coeliac disease
b.Cow’s milk protein allergy
c.Gastroenteritis
d.Intussusception
e.Lactose intolerance

A

Cow’s milk protein allergy – allergy symptoms and GI symptoms. If breast tell mum to avoid dairy. If bottle use hydrolyzed. Start on milk ladder after 1 year of age

114

Q

A 15 year old boy has 3 weeks of cough, weight loss, night sweats and fever. He moved to the UK from India 3 years ago. His chest x-ray is show in the image (can’t find exact image but showed some left upper zone shadowing I think). Which investigation is regarded as the gold standard for diagnosis for his underlying condition?

Blood culture

Gastric washings

Mantoux

QuantiFERON / IGRA

Sputum culture

A

Sputum culture

115

Q

and dark urine. Last week she finished a course of nitrofurantoin for a urinary tract infection. Her blood results are as follows: Hb 70 g/L (110-140), WBC 3.2 x 10^9/L (5.0 - 12.0), platelets 520 x 10^9/L (150 - 400), albumin 37 g/L (30 - 50), bilirubin 75 umol/L (<21), ALT 18 IU/L (0 - 29), ALP 130 IU/L (60 - 425). Which is the most likely underlying diagnosis?

Beta Thalassaemia

G6PD deficiency

Hereditary spherocytosis

Iron deficiency anaemia

Sickle cell disease

A

G6PD deficiency - Nitrofurantoin can cause it

116

Q

A 7 year old boy has a history of recurrent chest infections, persistent sinusitis and has been prescribed multiple courses of antibiotics. He has bibasal crepitations and a right-sided apex beat. Which is the most likely underlying diagnosis?

Agammaglobulinaemia

Cystic fibrosis

HIV

Primary ciliary dyskinesia – people commonly also have consanguity (left and right flipped inside)

Tracheo-oesophageal fistula

A

Primary ciliary dyskinesia (PCD)

This is because

Recurrent Respiratory Infections and Sinusitis: PCD is characterized by recurrent respiratory tract infections due to defective ciliary function, which impairs mucociliary clearance.

Bibasal Crepitations: These are common in conditions that affect the lower respiratory tract chronically, such as PCD.

Right-Sided Apex Beat (Situs Inversus): PCD is often associated with situs inversus (a condition where the major visceral organs are mirrored from their normal positions), which can result in a right-sided apex beat.

117

Q

A 3 day old baby girl is not feeding well. Her temperature is 36.7C, heart rate 180 bpm, respiratory rate 66 breaths per minute. There is a systolic murmur and femoral pulses are not palpable. Which is the most likely diagnosis?

Atrial septal defect

Coarctation of the aorta

Patent ductus arteriosus

Patent foramen ovale

Ventricular septal defect

A

Coarctation of the aorta

118

Q

. A 5 year old boy in the paediatric Emergency department recently had a sore throat and now has a rash across her lower legs. She is afebrile but has abdominal pain and joint pains inboth legs. Her full blood count result is as follows: Hb 110 g/L (115 - 140), WCC 10.3 x 10^9/L (3.8 - 10), neutrophils 6 x 10^9/L (150 - 400). Which is the most likely diagnosis?
a.Acute lymphoblastic leukaemia
b.Henoch Schonlein Purpura
c.Idiopathic thrombocytopenic purpura
d.Juvenile idiopathic arthritis
e.Meningococcal septicaemia

A

b.Henoch Schonlein Purpura

119

Q

A 15 year old boy presents to the Emergency Department with nausea, diarrhoea and abdominal pain, jaundice, lethargy and generalised joint pains. The diarrhoea is watery, pale in colour and does not contain blood. He returned from Thailand last week.
Investigations:
Hb 150 g/L (130 -175)
WBC 12.5 x 10^9/L (3.8 - 10.0)
Platelets 380 x 10^9/L (150 - 400)
Albumin 48 g/L (30 - 50)
ALT 650 IU (0-37)
ALP 280 IU (60 - 425)
Bilirubin 110 umol/L (<21)
Gamma-GT 94 IU/L (9 - 40)
Which is the most likely diagnosis?
a.Hepatitis A infection
b.Hepatitis B infection
c.Infectious mononucleosis
d.Shigella enteritis
e.Typhoid fever

A

Hepatitis A infection

120

Q

. An 8 year old boy in the Emergency Department has been unable to open his left eye for 1 day. Prior to this he had a cold one week ago. When examining the left eye, it is difficult to open, and he reports pain on all eye movements. The conjunctiva is injected. Which is the most appropriate next step in management?
a.Blood pressure
b.Blood tests
c.CT head
d.Lumbar puncture
e.Refer to specialist

A

Refer to specialist

NOTE: Do a CT head if you are between orbital and periorbital cellullitis

121

Q

A 4 year old boy has recently emigrated to the UK with a chronic persistent cough, frequent chest infections and loose stools. He has low weight, a wet cough and nasal polyps. His uncle had a similar condition, and died at age 28. Which investigation is discriminatory in this scenario?
a.Chest x-ray
b.Mantoux test
c.Pernasal swab
d.Sputum culture
e.Sweat test

A

Sweat test

122

Q

A 7 year old girl has breast buds but no pubic hair. Her height is on the 98th centile and her weight is on the 75th centile. She is otherwise fit and well. There is no significant family history. Investigations show: bone age of 10 years, FSH 3.2 IU/L (<1), LH 3.4 IU/L (<1). Which is the most likely diagnosis?
a.Adrenarche
b.Central precocious puberty
c.Congenital adrenal hyperplasia
d.Normal variant
e.Thelarche

A

Central precocious puberty

This is because:

Clinical Presentation:
Breast buds (thelarche): This indicates the beginning of puberty.
No pubic hair: Pubic hair development typically follows breast development in the sequence of puberty.

Growth Metrics:
Height on the 98th centile: Advanced growth may be associated with early puberty.
Weight on the 75th centile: Consistent with overall increased growth velocity.
Bone Age:

Bone age of 10 years: Advanced bone age suggests early onset of puberty.

Hormonal Findings:
Elevated FSH (3.2 IU/L) and LH (3.4 IU/L): Elevated gonadotropins are indicative of activation of the hypothalamic-pituitary-gonadal axis, which is a hallmark of central precocious puberty.

123

Q

A 4 year old boy is in the Emergency Department has 1 week of tiredness. His mother reports that he has generalised swelling, mostly around the eyes and lower legs. His abdomen also seems bigger than normal. Which is the most appropriate first line investigation in this scenario?
a.24 hour urine collection
b.Abdominal ultrasound
c.Abdominal x-ray
d.Blood tests
e.Urine dipstick

A

Urine dipstick - minimal change disease - check dipstick for nephrotic syndrome

124

Q

A 3 year old boy attends the Emergency Department after an episode of sudden onset jerky movements of both arms and legs for 5 minutes, which stopped before the ambulance arrived. His current observations are: temperature 38.5C, pulse 130 bpm, blood pressure 100/90 mmHg, respiratory rate 25 breaths per minute, oxygen saturation 98% breathing air. He has had a coryzal illness for 24 hours and is otherwise fit and well. He is alert, with a blood glucose of 4.5 mmol/L (3.0 - 6.0) and neurological examination is normal. Which is the most appropriate next step in management?
a.Order a CT brain and observe
b.Prescribe buccal midazolam and discharge
c.Prescribe IV ceftriaxone and admit
d.Prescribe oral antipyretics and observe
e.Prescribe oral co-amoxiclav and discharge

A

Prescribe oral antipyretics and observe

NOTE: Wouldn’t prescribe buccal midazolam. ALWAYS OBSERVE FIRST EPISODE OF CONVULSIONS. anti-pyretic for symptomatic relief.

125

Q

A term infant born to a diabetic mother is 4 hours old on the postnatal ward and is noted to be jittery. The capillary blood gas result is as follows: pH 7.37 (7.35 - 7.45), PCO2 5.5 kPa (4.6 - 6.4), PO2 4.9 kPa (8 - 12), glucose 2.4 mmol/L (3 - 6), lactate 2.0 mmol/L (1 - 2). Which is the most appropriate treatment for the baby in this scenario?
a.Buccal midazolam
b.Feed the baby
c.IV dextrose
d.IV lorazepam
e.Oxygen

A

b. Feed the baby

If feeding the baby does not resolve the hypoglycemia or if the infant is unable to feed, the next step would be to consider intravenous dextrose (option c). However, initial management typically involves an attempt to feed the baby.

126

Q

A 2 year old boy has acute onset of cough and drooling, with inspiratory upper airway noises. His vaccinations are up to date. His examination is otherwise normal. His temperature is 36.8C, pulse 120 bpm, blood pressure 100/60 mmHg, respiratory rate 26 breaths per minute and oxygen saturation 98% breathing air. Which is the most likely diagnosis?
a.Anaphylaxis
b.Croup
c.Epiglottitis
d.Inhaled foreign body
e.Laryngomalacia

A

d. Inhaled foreign body

Acute Onset of Symptoms: The sudden onset of cough and drooling is highly suggestive of an inhaled foreign body. This is often seen when a child suddenly develops respiratory symptoms without a preceding illness.

Inspiratory Upper Airway Noises: Stridor, which is an inspiratory noise, is commonly associated with an obstruction in the upper airway, which can occur with an inhaled foreign body.

Drooling: This is a key symptom that suggests the child is having difficulty swallowing saliva, which can be due to a partial obstruction in the airway or esophagus caused by a foreign body.

Normal Vital Signs and Examination: The child’s normal temperature and other stable vital signs make infectious causes like croup and epiglottitis less likely. Both of these conditions often present with fever and more severe respiratory distress.

Vaccinations Up to Date: This reduces the likelihood of epiglottitis, especially if the child has received the Haemophilus influenzae type b (Hib) vaccine, which significantly reduces the incidence of epiglottitis caused by Hib.

127

Q

A 14 year old girl attends Paediatric Outpatients with ongoing headaches. These are associated with nausea and vomiting and she is missing school. They are occurring once per month and at any time of day. The headaches eventually settle after lying down in a dark room. She has tried NSAIDs, which do not help. Neurological examination is normal. Which is the most appropriate treatment?
a.Aspirin once daily
b.Metoclopramide when required
c.Nasal sumatriptan when required
d.Pizotifen when required
e.Propanolol once daily

A

Nasal sumatriptan when required – it’s the normal treatment ladder, however after triptans should be managed by specialist; ask for abdominal migraine in children

128

Q

A 12 year old girl in GP complains of ongoing headaches for the last 4 months. The headaches are severe and throbbing in nature and often associated with nausea and vomiting. The headaches are worse in the morning. His father has noticed he is more tired than usual and has been more irritable. His weight is 62 kg. Which is the most likely diagnosis?
a.Benign intracranial hypertension
b.Insomnia
c.Low mood
d.Migraine
e.Tension headaches

A

a. Benign intracranial hypertension (also known as idiopathic intracranial hypertension)

Here’s the reasoning:

Headache Characteristics: Severe, throbbing headaches that are worse in the morning are a red flag for increased intracranial pressure (ICP).

Morning Headaches: Headaches that are worse in the morning suggest a possible increase in ICP because intracranial pressure is often higher when lying down.

Nausea and Vomiting: These symptoms are commonly associated with increased ICP.

Additional Symptoms: Increased tiredness and irritability can be related to increased ICP.

Weight: The girl’s weight (62 kg) may suggest obesity, which is a known risk factor for benign intracranial hypertension.

129

Q

  1. A 5 year old boy in General Practice has bright red blood when opening his bowels for 1 week. He opens his bowels every 3 days. He has no past medical history and is thriving. He has a well-balanced diet and has good fluid intake. There are no safeguarding concerns. Abdominal examination is unremarkable. Rectal inspection reveals a fissure. Which is the most appropriate management?
    a.Dietary advice
    b.Osmotic laxative
    c.Phosphate enema
    d.Stimulant laxative
    e.Toilet training

A

Offer an osmotic laxative as there is a fissure

130

Q

A 7 year old boy in General Practice has a sore throat, fever and pain on swallowing for 1 day. His temperature is 38.5C, pulse rate 110 bpm, respiratory rate 24 breaths per minute, and oxygen saturation 99% breathing air. He has bilateral tonsillar enlargement with exudate and bilateral cervical lymphadenopathy. He is talking in full sentences without using accessory muscles. Which is the most appropriate management?
a.Admit to hospital
b.Continue supportive measures
c.Prescribe amoxicillin
d.Prescribe phenoxymethylpenicillin
e.Review in 48 hours

A

D. Prescribe phenoxymethylpenicillin

NOTE: FeverPAIN criteria

Fever (during previous 24 hours)

Purulence (pus on tonsils)

Attend rapidly (within 3 days after onset of symptoms)

Severely Inflamed tonsils

No cough or coryza (inflammation of mucus membranes in the nose)

Each of the FeverPAIN criteria score 1 point (maximum score of 5). Higher scores suggest more severe symptoms and likely bacterial (streptococcal) cause. A score of 0 or 1 is thought to be associated with a 13 to 18% likelihood of isolating streptococcus. A score of 2 or 3 is thought to be associated with a 34 to 40% likelihood of isolating streptococcus. A score of 4 or 5 is thought to be associated with a 62 to 65% likelihood of isolating streptococcus.

131

Q

A 14 year old has a 2 day history of worsening abdominal pain, reduced appetite and constipation. His temperature is 38.1C. He is tender in his right lower quadrant. Which is the most appropriate action?
a.Admit to hospital for assessment
b.Encourage oral fluids
c.Prescribe analgesia and follow up next week
d.Prescribe movicol
e.Refer to paediatric 2 week wait

A

A. admit to hospital for assessment

THIS IS LIKELY APPENDICITIS

132

Q

A 2 year old in General Practice has a cough. He has reached all of his developmental milestones at the right time so far. Which motor skill would this child have most recently acquired?
a.Crawling
b.Draw a circle
c.Draw a square
d.Transfers objects from one hand to the other
e.Turning one page in a book

A

e. Turning one page in a book

Crawling (3 months head, 6 months upright, 9 months crawl, 12 months walk (18 is red flag))

Draw a circle (3 years)

Draw a square (4 years)

Transfers objects from one hand to the other (6 months)

Turning one page in a book (Several pages at 2 years, one page at 3 years) Casting bricks should stop by 18 months

133

Q

A 28 year old man attends the clinic with his mother. He states that he has ‘had enough’ because his flatmates are Russian spies who are after him, follow him wherever he goes and he can hear them comment on his actions when they are not in the room with him. As a result of this, he has stopped going out and has limited his contact with his family and friends in recent months. Which of the features described is a negative symptom?
a.He believes that he is being followed
b.He believes that his flatmates are Russian spies
c.He has limited social contacts
d.He hears the voices of his flat mates
e.His flatmates’ voices comment on his actions

A

c. He has limited social contacts

Negative symptoms in schizophrenia refer to deficits or reductions in normal emotional and behavioral functions. Limited social contacts indicate a reduction in social interactions, which is a negative symptom.

134

Q

  1. A 35 year old man has been feeling low in mood and has disturbed sleep. He has been taking fluoxetine 40 mg daily for 3 months, with no improvement. His treating doctor intends to commence him on citalopram 20 mg daily. Which is the most appropriate next step in regards to changing his medication?
    a.Add citalopram to fluoxetine at the current dose, withdrawing fluoxetine only if patient’s syndrome improve
    b.Cross taper, i.e. reduce the fluoxetine dose, simultaneously adding citalopram
    c.Reduce fluoxetine dose to 20mg daily, discontinue it after 1 week and then immediately initiate citalopram
    d.Reduce fluoxetine dose to 20mg daily and start citalopram after a 1 week washout period
    e.Reduce fluoxetine dose to 20 mg daily and start citalopram after a 6 week washout period

A

Option d. Reduce fluoxetine dose to 20mg daily and start citalopram after a 1-week washout period could indeed be a valid approach, especially considering the half-life of fluoxetine, which is around 4 to 6 days, and the potential risk of serotonin syndrome.

This approach allows for a gradual reduction of fluoxetine while minimizing the risk of adverse effects from the combination of SSRIs.

135

Q

An 18 year old woman has not eaten any solid food over the past week and has been limiting her liquid intake to 500 mL per day. She is preoccupied about being significantly overweight. Which blood test parameter is most likely to be raised in this scenario?
a.Cholesterol
b.Haemoglobin
c.Phosphate
d.Potassium
e.Triiodothyronine (T3)

A

Cholesterol: When the body is in a state of severe malnutrition and fasting, such as in this scenario, there can be alterations in lipid metabolism. Specifically, levels of cholesterol can increase due to the body mobilizing fat stores for energy. This can result in elevated serum cholesterol levels.

136

Q

A 46 year old woman attends a follow up appointment with the community mental health team. She has been taking venlafaxine 150mg daily for the past 3 months. She is feeling more energetic, requiring very little sleep, has been going out most evenings and is overspending. Which is the most appropriate next step in regards to her management?
a.Refer to the community mental health team
b.Start lithium
c.Start risperidone
d.Start sodium valproate
e.Stop venlafaxine

A

Stop venlafaxine

Venlafaxine: Venlafaxine is an antidepressant medication that belongs to the class of serotonin-norepinepherine reuptake inhibitors (SNRIs). It can induce or exacerbate manic or hypomanic symptoms, especially in individuals with bipolar disorder.

137

Q

A 20 year old woman attends the outpatient clinic following concerns by her GP about her eating habits. She significantly restricts her food intake and induces vomiting when she feels that she has overeaten. She speaks a lot about food and has started baking cakes, although she does not appear to eat them. She has not had a period for over 3 months. Her BMI is 18.5 kg/m2. She appears anxious when her diet is discussed. Which presenting feature most characteristically indicates a diagnosis of anorexia nervosa?
a.Amenorrhoea
b.Anxiety when discussing food
c.BMI of 18.5 kg/m2
d.Preoccupation with food
e.Purging behaviour

A

Amenorrhea: In females, the absence of menstrual periods (amenorrhea) is a common feature of anorexia nervosa, particularly when the disorder is severe and accompanied by significant weight loss. It typically occurs due to hormonal disruptions caused by malnutrition and low body weight.

138

Q

A 27 year old woman with diabetes reports not sleeping for 1 week. She has recently ended her relationship with her partner of 5 years because she believes that he is a member of the CIA and has been uncharacteristically promiscuous. She states that she hears his voice even when she is not with him in the room. Which medication combination is most appropriate in this scenario?
a.Aripiprazole and sodium valproate
b.Clozapine and lithium
c.Olanzapine and carbamazepine
d.Quetiapine and lithium
e.Risperidone and lamotrigine

A

Quetiapine and lithium antipsychotic and mood stabiliser. During depression the SSRI of choice is fluoxetine.

139

Q

A 35 year old woman is referred to the mental health nurse based in her GP surgery, following multiple presentations over the last year with gynaecological symptoms. She has a regular menstrual cycle, but with the onset of menstruation, she becomes increasingly anxious that she has developed life-threatening internal bleeding. She is worried that she may have cancer of the uterus. Blood tests, including a full blood count, are within normal limits. Her blood pressure is normal. Which is the most likely diagnosis?
a.Conversion disorder
b.Factitious disorder
c.Hypochondrial disorder
d.Malingering
e.Somatisation disorder

A

Hypochondrial disorder

140

Q

A 50 year old man has been feeling low in mood and describes ‘not enjoying anything’ in recent weeks. He has difficulties concentrating at work, where his performance has been declining. He is unkempt and losing weight. He has seen his GP who initiated sertraline 100mg daily 1 month ago and referred him to the community mental health team. Which is the most appropriate next step in his management?
a.Add lithium
b.Add mirtazapine
c.Add olanzapine
d.Increase the dose of sertraline
e.Switch to dosulepin (a TCA)

A

d.Increase the dose of sertraline

141

Q

A 25 year old woman is currently an informal inpatient on a mental health ward for management of a depressive episode. She has intent to end her life. She wants to leave the ward. She is deemed to have capacity to make a decision to leave hospital. Which is the next most appropriate step in management?
a.Allow her to self discharge
b.Detain her under the Deprivation of Liberty Safeguards
c.Detain her under the Mental Capacity Act
d.Detain her under the Section 2 of the Mental Health Act
e.Detain her under the Section 5(2) of the Mental Health Act

A

Detain her under Section 5(2) of the Mental Health Act

142

Q

. A 85 year old man in the memory clinic has concerns about being unable to recall what he ate for breakfast that morning or provide detail regarding his activities today and yesterday. He has difficulties concentrating. He has a tremor and poor balance. Which is the most appropriate initial step in his work-up?
a.Blood tests
b.CT brain
c.MoCA (Montreal Cognitive Assessment)
d.MRI brain
e.Neuropsychological testing

A

c.MoCA (Montreal Cognitive Assessment)

143

Q

A 22 year old woman in the Emergency Department describes an episode when she saw an unfamiliar man in her house at sunset. Her boyfriend, who was with her at the time, did not see this man in her house, but noted that she was disoriented to time and place. She reported no other difficulties at the time and she has no previous psychiatric history. Her AMTS is 6/10 and her temperature 37.9C. Investigations: GGT 11 IU/L (9 - 40), CRP 70 mg/L (<5); urine drug screen - negative for all substances. Which is the most likely diagnosis?
a.Alcoholic hallucinosis
b.Delirium
c.First episode psychotic disorder
d.Temporal lobe epilepsy
e.Young onset dementia

A

B. Delirium as she is unoriented to time and place

144

Q

A 30 year old woman is assessed at the addictions treatment clinic for her heroin addiction, having never had treatment previously. Which is the most appropriate pharmacological treatment to initiate?
a.Diazepam
b.Disulfiram
c.Methadone
d.Naloxone
e.Naltrexone

A

Methadone

145

Q

A 30 year old woman attends the Urgent Treatment Centre after cutting her wrists superficially. She suffers from depression and is taking sertraline 50mg daily. Her partner walked out on her this morning. She feels distressed but her sister has offered to stay with her. She has previously cut her wrists with no threat to her life. She has no thoughts of suicide. She intends to attend her appointment with the psychiatrist tomorrow. Which is the next most appropriate step in management?
a.Arrange a Mental Health Act assessment
b.Call an ambulance
c.Call the duty psychiatrist for an urgent assessment
d.Give her the 24 hour mental health crisis number
e.Increase her dose of sertraline

A

d.Give her the 24 hour mental health crisis number

146

Q

A 24 year old paramedic is referred to see a psychiatrist by her GP. She reports that she has not been herself after attending a call last month when the patient ultimately died. She complains of low mood and difficulty falling asleep on most nights. Her appetite is not affected and she has not lost any weight. Blood test results including full blood count and thyroid function tests are unremarkable. Which is her most likely diagnosis?
a.Acute stress reaction
b.Adjustment disorder
c.Depressive episode
d.Generalised anxiety disorder
e.Post-traumatic stress disorder

A

Acute stress reaction

147

Q

A 44 year old man has a 10 year history of using crack cocaine and heroin. He uses both substances once or twice per week and often feels depressed following episodes of excessive use. He developed hepatitis C after sharing needles. He does not describe a strong desire to use these substances, but nonetheless continues to do so. Which diagnosis best describes his presentation?
a.Acute intoxication
b.Dependence
c.Depressive disorder
d.Harmful use
e.Withdrawal syndrome

A

Harmful use – substance use causing harm to health

Dependence – strong desire to take substance
Withdrawal syndrome – symptoms result after stopping substance

148

Q

A 55 year old woman in General Practice has a history of recurrent depression. She has previously had cognitive behavioural therapy and antidepressant medication. She is currently well but her work has been stressful and she is worried her depression may recur. Which is the most appropriate next management option?
a.Group based CBT
b.Mindfulness-based cognitive therapy
c.Prescribe prophylactic SSRI
d.Prescribe zopiclone
e.Regular follow up with GP and screening for depression

A

Mindfulness-based cognitive therapy – this can be used for prevention

149

Q

A 55 year old man attends for a routine health review. He was seen on a few occasions last year after he was made redundant from his job as an IT technician. At the time he had low mood, anhedonia, insomnia and hopelessness, which all resolved after he found a new job. Which is the most likely diagnosis?
a.Adjustment disorder
b.Bipolar affective disorder
c.Personality disorder
d.Post-traumatic stress disorder
e.Severe depressive disorder

A

Adjustment disorder

150

Q

A 33 year old woman takes omeprazole, paroxetine, aripiprazole, and zopiclone for gastro-oesophageal reflux and bipolar affective disorder. She also takes the combined oral contraceptive pill. Her annual blood test show: sodium 135 mmol/L (135 - 145), potassium 4.2 mmol/L (3.5 - 5.3), urea 5.1 mmol/L (2.5 - 7.8), creatinine 63 umol/L (60 - 120), eGFR 90 (>60) and prolactin 622 UI/L (100 - 500). Which medication is most likely responsible?
a.Aripiprazole
b.Combined oral contraceptive pill
c.Omeprazole
d.Paroxetine
e.Zopiclone

A

a.Aripiprazole

151

Q

A 39 year old woman, gravidity 2 parity 0 at 39 + 3 weeks’ gestation is on the labour ward. She presented in spontaneous labour with a history of regular contractions every 4-5 minutes, lasting 30 seconds. Her MEWS (maternity early warning score) is 0. The foetal heart rate is reassuring. At 09:00 she was 2 centimetres dilated on vagin*l examination. At 13:00 she was 5 centimetres dilated and at 17:00 she was 6 centimetres dilated. What is the most appropriate next step in management?
a.Artificial rupture of membranes
b.Commence oxytocin
c.Perform a caesarian section
d.Perform a membrane sweep
e.Wait for 2 hours and reassess

A

Artificial rupture of membranes – this is the first line in failure to progress (if this doesn’t work then needs to be obstetric-led care). Next step or if membranes are already ruptured, oxytocin (Failure to progress is <2cm in 4 hours)

NOTE: Perform a membrane sweep – first step for induction of labour (done around 40 weeks), followed by vagin*l prostaglandin, followed by oral, followed by oxytocin

152

Q

. A 24 year old woman in her first pregnancy, is in spontaneous labour on the labour ward. She is using Entonox for analgesia and has now been in the second stage for 3 hours. She has been in good descent from pushing for 2 hours. The baby is in the cephalic presentation in an occipito-anterior position, vertex below the ischial spines. The foetal heart rate has been suspicious for 30 minutes. Which is the most appropriate management in this scenario?
a.Caesarean section
b.Epidural analgesia
c.Episiotomy
d.Forceps delivery
e.Ventouse delivery

A

Episiotomy – this is going to be done first to make instrumental delivery easier

Forceps delivery – need to expedite the delivery since its been 2 hours and signs of fetal distress. This is quicker then ventouse

153

Q

A 39 year old woman has a transvagin*l ultrasound scan at 6 weeks of pregnancy, in which an intrauterine gestational sac was seen. She attends the Emergency Department 2 weeks later with 2 days of heavy vagin*l bleeding. A transvagin*l scan shows a normal endometrium. Both ovaries appear normal. There is no adnexal mass but there is a small amount of free fluid in the Pouch of Douglas. Which is the most likely diagnosis?
a.Complete miscarriage
b.Ectopic pregnancy
c.Incomplete miscarriage
d.Molar pregnancy
e.Pregnancy of unknown viability

A

Complete miscarriage

154

Q

A 42 year old woman is 12 weeks into her third pregnancy. She would like to have screening for Trisomy 21. Which is the most sensitive screening test?
a.Amniocentesis (99.4-100%)
b.Chorionic villous sampling (85%)
c.Non-invasive pre-natal testing (cell free DNA) (99.17-100%)
d.Nuchal translucency
e.Quadruple test

A

Amniocentesis – this is actually most sensitive >15 weeks

Chorionic villous sampling - less sensitive done around 11-14 weeks

Non-invasive pre-natal testing (cell free DNA) –slightly less sensitive, is now on the NHS

NOTE: Depends on question, think about risk to baby, mothers wishes etc

155

Q

A 27 year old woman in the early pregnancy unit has light vagin*l bleeding, 6 weeks after her last menstrual period. Transvagin*l ultrasound scan (USS) shows an intrauterine gestation sac. There was no visible yolk sac or foetal pole. Which is the recommended management?
a.Arrange a repeat USS in 10-14 days
b.Arrange a repeat USS in 48 hours
c.Offer management for miscarriage
d.Reassure and discharge
e.Take beta-HCG and repeat in 48 hours

A

Arrange a repeat ultrasound scan (USS) in 10-14 days

Here’s why:

Intrauterine Gestation Sac: The presence of an intrauterine gestational sac indicates a pregnancy is located within the uterus, which is reassuring.

Absence of Yolk Sac or Fetal Pole: At 6 weeks of gestation, the absence of a visible yolk sac or fetal pole may not be definitive evidence of a non-viable pregnancy. In some cases, these structures may not yet be visible due to the early stage of pregnancy.

156

Q

A 38 year old woman has a total abdominal hysterectomy for dysmenorrhoea. Histology shows that there were CIN 1 changes in the cervix, which were completely excised. Which is the most appropriate follow-up?
a.Back to routine smear recall
b.Colposcopy in 6 months
c.No follow up required
d.Vault smear in 6 months
e.Vault smear in 12 months

A

No follow up required

157

Q

  1. A 35 year old woman has a severe headache 24 hours following an elective cesarean section for placenta praevia under regional anaesthesia. It is worse on sitting up and better when she is lying down. Her temperature is 37.3C, heart rate 90 bpm, blood pressure 124/64 mmHg, respiratory rate 12 breaths per minute, oxygen saturation 98% breathing room air. Which is the most likely cause of her headache?
    a.Cerebral venous sinus thrombosis
    b.Meningitis
    c.Migraine
    d.Post-dural tap headache
    e.Pre-eclampsia

A

Post-dural tap headache - side-effect of epidural, headache that is worst when standing and better on lying flat. Get them to lie flat as long as possible and paracetamol. If persisits epidural blood patch

158

Q

A woman sees her GP at 12 weeks gestation. Which vaccines should she be recommended?
a.BCG and pertussis
b.Influenza and pertussis
c.Influenza and rubella
d.Pertussis and varicella zoster
e.Rubella and varicella zoster

A

Influenza and pertussis - (+COVID-19), also tested for HIV, Hep B and Syphillis.

159

Q

A 22 year old woman with cystic fibrosis attends the obstetric medicine clinic for preconception counselling. Development of which obstetric complication is she at greatest risk, given her pre-existing disease?
a.Foetal congenital malformation
b.Gestational diabetes
c.Obstetric cholestasis
d.Pregnancy induced hypertension
e.Spontaneous miscarriage

A

Gestational diabetes – you are at increased risk for diabetes anyways

160

Q

A 24 year old vegan woman with alpha thalassemia trait has a booking haemoglobin of 115 g/L (115 - 150), WCC 7.9 x 10^9/L (3.8 - 10), platelets 230 x 10^9/L (150 - 400). At her 28 week appointment her Hb is 106 g/L, WCC 8.9 x 10^9/L, platelets 195 x 10^9/L. Which is the most likely cause for these test results?
a.Alpha thalassaemia
b.Diet
c.Myeloproliferative disorder
d.Physiological change
e.Retroplacental bleeding

A

Physiological change

161

Q

A 20 year old woman has abdominal discomfort for 3 months. A pregnancy test is negative. An ultrasound scan shows a complex cystic pelvic mass with solid components. Which is the most likely diagnosis?
a.Endometriosis
b.Epithelial ovarian cancer
c.Fibroids
d.Germ cell tumour of the ovary
e.Uterine sarcoma

A

d. Germ cell tumor of the ovary

Germ cell tumors of the ovary are neoplasms derived from germ cells and can present as complex cystic masses with solid components on ultrasound. These tumors are more common in young women, typically in their second or third decade of life. While they can be benign or malignant, the presence of solid components within the cystic mass raises concern for malignancy.

162

Q

A 10 year old girl with BMI 28 kg/m2 is seen in adolescent gynaecology outpatients. She has breast bud development and some axillary hair. There are no other features of secondary sexual characteristics or galactorrhoea. Which is the most appropriate next step in management?
a.Follow-up in 2 years
b.Hormone profile
c.MRI brain
d.Pelvic ultrasound scan
e.Reassure and discharge

A

Reassure and discharge – order of puberty breast buds, pubic hair, menarche; for boys testis, penis, pubic hair voice

163

Q

. A 48 year old woman has a progesterone IUS in place. She complains of a 1 year history of palpitations, night sweats, fatigue, irritability and vagin*l dryness. What is the most appropriate treatment?
a.Oral oestrogen
b.SSRIs
c.Tibolone
d.Transdermal oestrogen
e.vagin*l oestrogen

A

Transdermal oestrogen

164

Q

A 55 year old woman in gynaecology outpatients has a BMI of 23.5 kg/m2 and a history of stress incontinence. Pelvic floor exercises have not improved her symptoms. Which is the most appropriate next step in management?
a.Hysterectomy
b.Intra urethral bulking
c.Ring pessary
d.Surgical repair
e.Weight loss

A

Surgical repair

NOTE: Intraurethral bulking (option b) is often considered before surgical repair because it is a minimally invasive procedure that can be effective for treating stress incontinence and carries fewer risks compared to surgery. If intraurethral bulking fails to adequately address the symptoms, then surgical repair may be pursued

165

Q

A 34 year old woman has an evacuation of retained products of conception, which are sent for cytogenetics. She is told she had a molar pregnancy. Which is the most appropriate follow up?
a.Chest x-ray
b.CT pelvis
c.Serial pipelle endometrial biopsy
d.Serial quantitative beta-HCG
e.Serial ultrasound scans

A

d.Serial quantitative beta-HCG

166

Q

A 23 year old woman has superficial dyspareunia and type 3 female genital mutilation. Which is the most appropriate next step in management?
a.Make a safeguarding risk assessment
b.Offer reversal surgery
c.Refer for psychosexual counselling
d.Refer to police
e.Refer to social services

A

a.Make a safeguarding risk assessment

167

Q

56 year old woman has a 6 month history of urinary incontinence and urgency, which is affecting her personal life. Urine microscopy is negative. Which is the most appropriate first investigation?
a.Bladder diary
b.Cystoscopy
c.Pad tests
d.Pelvic ultrasound
e.Urodynamics

A

Bladder diary

Here’s why:

Bladder Diary: A bladder diary is a simple and non-invasive tool used to assess the frequency, volume, and pattern of urinary symptoms over a specific period. It typically involves recording fluid intake, voiding frequency, urine volume, episodes of urinary incontinence, and any associated factors such as urgency or nocturia. A bladder diary can provide valuable information about the nature and severity of the patient’s symptoms, as well as potential triggers or patterns.

168

Q

. A 23 year old woman complains of severe dysmenorrhoea every month for the last 6 months, lasting 3-4 days after her period ends. It is not relieved by regular paracetamol and a hot water bottle. Which is the most appropriate next step in management?
a.List for laparoscopy
b.Organise a pelvic CT scan
c.Organise a pelvic ultrasound scan
d.Perform a hormone profile
e.Perform a sexual health screen

A

c.Organise a pelvic ultrasound scan

169

Q

A 19 year old woman presents with an acutely painful large swelling near the opening of the vagin*. She is unable to sit and passing urine is uncomfortable. Her last menstrual period started 2 days ago. Which is the most likely diagnosis?
a.Bartholin’s abscess
b.Bartholin’s cyst
c.Congenital urethral cyst
d.Urinary tract infection
e.vagin*l endometriosis

A

Bartholin’s abscess

170

Q

A 57 year old woman in General Practice has vulval itching and passes urine more frequently. She denies any vagin*l discharge. The vulval area looks dry with evidence of excoriation marks. There is an area of paler skin near the introitus. Urinalysis showed 1+ leukocytes only. Which is the most likely diagnosis?
a.Atrophic vaginitis
b.Lichen sclerosus
c.Urinary tract infection
d.vagin*l candida
e.vagin*l malignancy

A

Lichen sclerosus – atrophy of epidermis with white patches, presents with itching and pain during sex/urination

171

Q

. A 34 year old woman attends her GP for her 32 week antenatal appointment. She feels well in herself and has regular foetal movements. She has had normal scans to date and an uncomplicated pregnancy. Her temperature is 36.8C, pulse 93 bpm, blood pressure 113/72 mmHg, and oxygen saturation 98% breathing air. Her booking blood pressure was 110/68 mmHg.

Urinalysis - negative
Ketones - negative
Blood - negative
Protein - 2+
Nitrites - negative
Leukocytes - negative
Which is the most appropriate next step in management?
a.Ask midwife to review in 2 weeks
b.Reassure and review at 36 week antenatal appointment
c.Refer for antenatal same day assessment
d.Send a mid-stream urine for microscopy, culture and sensitivity
e.Send a protein creatinine ratio

A

Refer for antenatal same day assessment - 2+ protein needs urgent same day assessment. 1+ and other symptoms of pre-eclampsia same day assessment as well

NOTE: Send a protein creatinine ratio – do this 1 week later if it was 1+ and no other symptoms

172

Q

A 30 year old woman presents to General Practice 2 weeks after delivery of her first baby. She has low mood, exhaustion, difficulty sleeping, and is very tearful for the last week. She felt well during her pregnancy. She is bonding well with her baby and has good support at home. She has a history of depression but has been stable for the last 2 years. She has no thoughts of self-harm or suicide. Which is the most likely diagnosis?
a.Adjustment disorder
b.Baby blues
c.Dysthymia
d.Postnatal depression
e.Recurrent depression

A

Postnatal depression

173

Q

A 35 year old woman in General Practice is 10 weeks pregnant and has worsening nausea and vomiting daily for 3 weeks. She is now struggling to keep down fluids but still passing urine.

Urinalysis:
Glucose - negative
Protein - negative
Ketone - negative
Blood - negative
Nitrites - negative
Leukocyte - negative
Which is the most suitable management option?
a.Admit to hospital
b.Advise ginger tea
c.Avoid heavy meals
d.Offer cyclizine
e.Offer ondansetron

A

Admit to hospital

NOTE: Offer cyclizine – do this if she can swallow. Offer ondansetron – this is second line as associated with increased risk of cleft palate.

174

Q

A 21 year old woman in General Practice has a 6 month history of irregular periods. She has gained 3 kg in weight over the last 4 months. Her BMI is 28 kg/m2 and she has cystic acne on her back. Which blood tests could support the most likely diagnosis?
a.Cortisol, fasting glucose
b.HbA1C, fasting lipids
c.LH, FSH, SHBG, testosterone
d.Prolactin, growth hormone
e.TSH, free T4

A

c.LH, FSH, SHBG, testosterone

175

Q

A 42 year old woman in General Practice has 6 weeks of intermenstrual and post-coital bleeding. She is married with two children. She has no vagin*l discharge. She has not previously taken hormonal contraception. Her last smear was normal 2 years ago. Which is the most likely diagnosis?
a.Cervical cancer
b.Endometrial cancer
c.Endometrial polyp
d.Ovarian cancer
e.Sexually transmitted infection

A

Endometrial polyp

176

Q

A 35 year old woman in General Practice has epilepsy and is currently taking sodium valproate and desogestrel. She has not had a seizure for 13 months. She and her boyfriend would like to conceive a child. Which is the most appropriate management option?
a.Perform blood tests to check sodium valproate levels
b.Reduce sodium valproate dose
c.Refer to epilepsy clinic, continue current medication in the meantime
d.Refer to epilepsy clinic, meanwhile switch to a different anti-epileptic
e.Stop all medication now and refer to epilepsy clinic

A

Refer to epilepsy clinic, continue current medication in the meantime – you do not want to change any of her medication, only epilepsy clinic can do this (sodium valporate teratogenic)

177

Q

An 8 month old baby is seen in clinic with widespread rash localised to his flexures, cheek and neck. There is a strong family history of atopy. What is the most appropriate first line treatment?

A

Emollients, topical steroids if severe

178

Q

A widespread rash is found on a 2 day old baby at his newborn check (see image). He is feeding well, is active and has no fever. What is the most appropriate management?

A

Milia (LOOK UP WHAT THIS LOOKS LIKE) - do nothing

179

Q

A 9 month old boy in the Paediatric Emergency Department has left cheek bruising. He had multiple bruises on both thighs and buttocks. Full blood count and clotting screens were unremarkable. What is the most likely cause for his presentation?

A

Non-accidental injury

180

Q

A 6 year old girl with known sickle cell disease is in the Emergency Department with lethargy and shortness of breath. This is in the context of a recent febrile illness with a rash that has now resolved. Her blood results show: Hb 45 g/L (115 - 140), WBC 2.8 x 10^9/L (3.8 - 10), platelets 55 x 10^9/L (150 - 400). What pathogen is likely to have caused this presentation

A

Parvovirus B19

181

Q

A 6 week old girl in the Paediatric Emergency Department has jaundice which has been present since 1 week of age. She has pale stools and dark urine. She has dropped two centiles for weight. What is the most likely diagnosis?

A

Biliary atresia

182

Q

A 12 year old girl develops discolouration of her urine and tears after starting medication for persistent chronic cough and fever. What medication commonly used for this condition is causing her symptoms?

A

rifampicin

183

Q

A 6 year old girl in clinic has asthma, and is currently on treatment with salbutamol and Clenil inhalers. What device is most suitable for delivering these medications?

A

spacer

184

Q

A small for gestational age baby is born following an unremarkable pregnancy. He has low set ears, micrognathia, microcephaly, rocker bottom feet, overlapping digits, cleft lip and palate. What is the most likely diagnosis following chromosomal analysis?

A

Trisomy 18

185

Q

A 5 month old girl in General Practice has a worsening nappy rash for the last 2 weeks despite regular nappy changes, barrier cream and nappy-free time. She is feeding normally and otherwise well. Her temperature is 36.8C, and pulse rate 128 bpm. The rash is depicted in the image provided. What is the most appropriate management option in this scenario?

A

mild steroid eg hydrocortisone, imicazole if this is candida

186

Q

A 30 year old man with a brain injury following a road traffic accident reports experiencing an odd smell of burnt plastic. He is surprised that this is not bothering his flatmates. He recently reported seeing an image of Jesus on the walls of his bedroom reciting the ten commandments. What is the most likely diagnosis?

A

Post-traumatic psychosis

Post-traumatic psychosis can occur following a traumatic brain injury and is characterized by the presence of hallucinations, delusions, disorganized thinking, or other psychotic symptoms. These symptoms may develop weeks to months after the injury and can be distressing and impairing for the individual.

187

Q

A 17 year old girl in the Emergency Department has experienced a seizure. She appears confused and fatigued, and has an irregular heart rhythm. Her blood pressure is 170/90 mmHg. Her mother reports that she has anorexia nervosa and her father has been trying to force feed her. What diagnosis would most likely explain her clinical presentation?

A

Refeeding syndrome - hypophosphotaemia

188

Q

A 56 year old man is brought to the Emergency Department by ambulance after he was found collapsed in a park. His pupils are constricted and his respiratory rate is 6 breaths per minute. What emergency medication should be administered in this scenario?

A

Nalaxone

189

Q

A 21 year old man with a history of drug use is detained under the Mental Health Act (1983) for treatment of an acute psychotic episode. He has been administered an initial dose of zuclopenthixol depot 2 days ago. His urine drug screen is positive for cannabis. He is confused, has muscle rigidity, fever and hyporeflexia. His temperature is 38.4C, pulse rate 113 bpm, BP 135/80 mmHg, respiratory rate 26 breaths per minute and oxygen saturation 96% breathing air. What is the most likely cause for the change in presentation?

A

Neuroleptic malignant syndrome

190

Q

A 75 year old man with recurrent depressive disorder stopped taking his medication 1 month ago. He now has significant low mood and suicidal thoughts. He is admitted informally following an apparent attempt to hang himself. His wife died of cancer 2 years ago and he has no children. What level of observation should be provided by the ward nursing staff in this scenario?

A

1:1

191

Q

A 30 year old woman is admitted to the Mental Health Unit. She has reportedly been spending her savings on expensive items of clothing. Her named nurse reports that she hardly sleeps at night and seems unable to settle down. She is overfamiliar and labile in mood. A urine drug screen is negative for all substances and all blood test parameters are within normal limits. What is the likely diagnosis?

A

Manix episode, This could be hypomania if she is functional.

192

Q

A 76 year old man who lives independently is admitted to the mental health unit with a severe depressive episode. He was managed in the community with multiple antidepressants at various times, but none have really benefited him. He completed a course of ECT treatment a few days ago. His family members visit him on the ward but he is unable to recognise them. What type of memory impairment is demonstrated?

A

Retrograde amnesia

193

Q

A 30 year old male with paranoid schizophrenia has been treated unsuccessfully with aripiprazole and later, olanzapine. He has ongoing delusions that his family and nursing staff are poisoning his food and medication. He hears two men arguing and commenting on his actions. What is next most appropriate pharmacological treatment option?

A

Clozapine - for treatment resistant schizo

194

Q

A 35 year old woman who is 7 weeks postpartum, presents with low mood, anhedonia, poor energy levels, insomnia and fleeting suicidal thoughts. She has no previous history of mental illness. She is breastfeeding. Initiation of what medication class is most appropriate in this scenario?

A

SSRI - sertraline and paroxetine given in breastfeeding

195

Q

A 61 year old man with alcohol dependence is brought to the Emergency Department by ambulance following concerns by neighbours. He presents with confusion and disorientation. He is irritable, tremulous and sweating. He appears to be responding to external visual stimuli. His temperature is 37.9C, pulse rate 125 bpm, BP 140/95 mmHg, respiratory rate 20 breaths per minute and oxygen saturation 95% breathing air. His breathalyser reading is 80 microgram alcohol / 100 mL of breath (legal limit for drivers <35). What is the most likely diagnosis?

A

Delirium tremens

196

Q

A 13 year old boy attends the child psychiatry clinic with his parents, who describe a 7 month history of overeating. He frequently rummages through the cupboards at odd times of the day. He has said at times that he feels fat. His BMI is 18.5 kg/m2 (just under the 50th centile for his age). He has calluses on his knuckles. He performs well academically. What is the most likely diagnosis?

A

Bulimia nervosa

197

Q

A 20 year old medical student has become increasingly isolated and has stopped attending lectures for the past 7 months. Concerns have been raised by his friends that he has been locking himself in his room and talking to himself. He believes that the government is spying on him and trying to steal his intelligence. He reports hearing female voices commenting on his actions. What is the most likely diagnosis?

A

Paranoid schizophrenia

198

Q

A 20 year old woman in the outpatient clinic has sudden episodes of palpitations, shortness of breath, sweating, chest pain and dry mouth. These episodes started 6 months ago, occur randomly and last for about 15 minutes. Education and lifestyle modification advice have not been helpful. What is the most appropriate next step in regards to her management

A

CBT or SSRIs can be used in panic disorder or GAD after self help/education

199

Q

A 20 year old man has never been able to hold down a job for longer than a few months at a time. His relationships have also been affected by his symptoms. He has recently been diagnosed with ADHD by his psychiatrist. What medication would the psychiatrist most likely recommend in this scenario?

A

Methylphenidate (lisdexamphetamine is second line, both are cardiotoxic so do baseline ECG)

200

Q

An 18 year old girl in General Practice has had no periods for the previous 4 months. She had the contraceptive implant inserted 2 years ago. She reports missing meals as she has been stressed for her university exams. She admits to making herself vomit occasionally when she is stressed. Her temperature is 36.8C, pulse rate 65 bpm, BP 109/70 mmHg, BMI 17.5 kg/m2. A pregnancy test is negative. What is the most likely diagnosis?

A

Anorexia nervosa

201

Q

A 55 year old woman in General Practice is in a tearful state and reports feeling low. She advises that she gets cold easily and has been gaining weight. What investigation will likely be discriminatory in this scenario?

A

TFTs can cause depression, cold and weight gain

202

Q

A 28 year old woman with a twin IVF pregnancy has a scan at 18 weeks gestation. This shows a marked difference in foetal size with oligohydramnios in one twin and polyhydramnios in the other twin. What is the most likely diagnosis

A

Twin to twin transfusion syndrome - treated with a tertiary care laser to stop blood supply

203

Q

A 36 year old woman presents with spontaneous rupture of membranes at 33 weeks’ gestation. She has no abdominal pain. Speculum examination confirms a <1cm dilated cervix. She has no allergies. Her temperature is 36.6C, pulse 88 bpm, BP 110/55 mmHg, respiratory rate 16 breaths per minute, oxygen saturation 99% breathing air. What antibiotic should be prescribed in this scenario?

A

Erythromycin, would give benpen if GBS confirmed or fever during labour

204

Q

An active 50 year old who has no medical problems has a total abdominal hysterectomy for large multiple fibroids. She loses 300mls of blood. The procedure is uncomplicated. For how long should she be advised to take bed rest following discharge home?

A

4-6 weeks

205

Q

A 36 year old woman with a long history of dysmenorrhoea and cyclical rectal bleeding presents with worsening abdominal pain. Regular paracetamol has not eased her symptoms. She has been trying to conceive for 8 months. Her LMP was 3 weeks ago. What is the most likely diagnosis?

A

Endometriosis, pain on defecation is due to dyschezia

206

Q

A woman is admitted at 12 weeks with severe nausea and vomiting. She requires three different anti-emetic treatments and IV fluids. What supplement must also be prescribed in this scenario?

A

Thiamine

207

Q

A 33 year old woman is seen at 7 weeks of pregnancy for booking. She has a history of ectopic pregnancy and salpingectomy, two first trimester miscarriages, a miscarriage at 23+5 weeks’ gestation, two failed IVF cycles and a premature delivery at 27 weeks’ gestation. The infant died of complications related to prematurity four weeks later. What is her gravidity and parity? (Your answer should be expressed in the format: GX PX

A

G5P1 (Gravidity is number of pregnancies and Parity is number of viable pregnancies >24 weeks)

208

Q

A woman is due to have an elective caesarean section for breech presentation at 39 weeks’ gestation. For how many hours should she be fasted for solids prior to the operation? Your answer should be expressed numerically in the form: X hours

A

6 hrs food and 2 hrs clear liquids

209

Q

A 27 year old woman has a non-viable pregnancy at 6 weeks with a beta-HCG of 28,000 IU/L. What malignancy is she at greatest risk of developing?

A

Choriocarcinoma

210

Q

A midwife is on her own with a patient on the labour ward. She states that the head has been delivered for 4 minutes but the body is not coming. The woman is lying flat on her back at the end of the bed. Name the first manoeuvre that should be performed in this scenario?

A

McRoberts’s maneuvere

211

Q

A 19 year old woman complains of irritability, aggression and low mood every month. She has no other medical conditions, takes no regular medication and is sexually active. The symptom diary shows that her work and social life are being affected in a cyclical way. What, specifically, is the aim of the hormonal management in this scenario?

A

Reduce PMS symptoms

212

Q

A 27 year old woman has a complete molar pregnancy and only receives surgical management. She subsequently has regular serum beta-HCG monitoring. For how long should she be told to avoid pregnancy after the beta-HCG has returned to normal? (Your answer should be expressed numerically in the form: X months)

A

6-12 months

213

Q

A 65 year old woman presents with a lump in her vagin*, which has been getting worse over many years. She finds it difficult to evacuate her bowels and now needs to digitate her vagin* to do so. What is the most likely diagnosis?

A

Rectocele

214

Q

A 24 year old woman with a BMI of 22 kg/m2, has an elective, uncomplicated laparoscopic procedure for removal of a 6 cm simple ovarian cyst. Assuming good postoperative recovery, after how long would she be appropriate to be discharged? (Your answer should be expressed numerically in the form: X hours

A

6 hours

215

Q

A 30 year old woman in the third trimester of her first pregnancy develops an itchy, bumpy rash on her abdomen, with sparing of the periumbilical area. She is usually fit and well and has had an uneventful pregnancy so far. What is the most likely diagnosis?

A

Pruritic urticarial papules and plaques of pregnancy (PUPPP)

216

Q

A baby girl is seen for her 6 week baby check in General Practice, with resolving yellow discolouration of the skin. She was born at term and is following the 25th centile for height and weight. She is breastfeeding well and stools are normal. What is the most likely diagnosis?

A

Physiological jaundice presents around day 3-4 and resolves by week 2. Above week 2 is prolonged jaundice. Since he is good most likely thing is breast milk jaundice

217

Q

A 25 year old woman is having her first smear in General Practice. The nurse notices an absent cl*tor*s and labia minora with surrounding scarring. What is the most likely diagnosis?

A

T2FGM

PPQ Flashcards by ROHIT LAWRENCE (2024)
Top Articles
Latest Posts
Article information

Author: Ms. Lucile Johns

Last Updated:

Views: 6067

Rating: 4 / 5 (61 voted)

Reviews: 84% of readers found this page helpful

Author information

Name: Ms. Lucile Johns

Birthday: 1999-11-16

Address: Suite 237 56046 Walsh Coves, West Enid, VT 46557

Phone: +59115435987187

Job: Education Supervisor

Hobby: Genealogy, Stone skipping, Skydiving, Nordic skating, Couponing, Coloring, Gardening

Introduction: My name is Ms. Lucile Johns, I am a successful, friendly, friendly, homely, adventurous, handsome, delightful person who loves writing and wants to share my knowledge and understanding with you.